Exam

Réussis tes devoirs et examens dès maintenant avec Quizwiz!

psychodynamic

Which frame of reference suggests that unresolved childhood events are the reason for dysfunction. A psychodynamic intervention is usually discussion based.

2.4 (aimless walking)

Which Allen Cognitive level is indicative of aimless walking: 24-hour nursing care to initiate and assist with all activities of daily living and to prevent wandering and getting lost.

cognitive disability

Which frame of reference uses the client's strengths to allow for function. An example of a cognitive disability intervention is training caregivers to provide appropriate environmental supports for the client.

A (The Self-Directed Search measures a client's vocational interests.)

A client with developmental coordination disorder is referred for evaluation by the transition services team at a high school. Which assessment is the MOST appropriate for the OTR® to complete first with the client? A. Self-Directed Search B. Cognitive Status Examination C. Box and Block Test D. Functional capacity evaluation

B

A client with multiple sclerosis (MS) presents with extensor tone in the bilateral lower extremities, which interferes with work tasks completed at a desk. The OTR® performs a worksite evaluation. What would the OTR® recommend to decrease spasticity? A. Trialing moist heat pads B. Positioning the hips into 90° or more of flexion. C. Taking frequent rest breaks to stretch D. Using a reclining chair to open the hips

B (Children younger than age 12 are usually placed on an immobilization protocol because of their low maturity level and low ability to comply with the exercises and precautions of other protocols.)

A 10-year-old child sustained a Zone 2 flexor digitorum profundus tendon laceration and underwent primary repair. Which flexor tendon repair protocol represents best practice standards? A. Active mobilization approach B. Immobilization approach C. Passive mobilization approach D. Controlled early active mobilization approach

B (Research indicates that proprioceptive input inhibits vestibular stimulation.)

A 4-year-old child with attention deficit hyperactivity disorder is using a tire swing for vestibular input to feel more organized. After a few minutes, the child appears to have some autonomic reactions to the fast swinging. Which course of action for the OTR® is BEST? A. Immediately ask the child to stop and sit in a corner to relax B. Ask the child to slow down the swinging by pulling a trapeze bar for proprioceptive input C. Ask the child to slow down while doing slow, deep breathing D. Let the child continue because the vestibular stimulation ceases with autonomic activation

C (This syndrome is associated with damage to the large nerve fibers that carry information directly from the cerebral cortex to the spinal cord. These nerves are particularly important for hand and arm function. )

A 70 year old patient is admitted into the acute care unit of a hospital secondary to loss of of fine motor control in the arms and hands and weakness in the legs. Upon initial evaluation, an OTR observes that the patient is able to not able to move the hands at all, but has some shoulder and elbow movement. What is the likely cause for this impairment? A. Brown Sequard B. Anterior cord syndrome C. Central cord syndrome D. Posterior cord syndrome

Left sided CVA

A patient in acute care demonstrates difficulty with visual verbal processing and bilateral motor praxis. She has difficulty processing images on the right side. Her symptoms are reflective of a l______________ s______________ C________

D (Access the bathroom using a wheelchair is often challenging. A bedside commode provides an alternative means for toileting.)

A 76-year-old client will be using a wheelchair after discharge from an acute rehabilitation facility. The client has achieved independence in wheelchair mobility on level surfaces but still requires minimal assistance for transfer. The client is planning to move into a daughter's home, which was not the client's previous residence. The OTR® conducts an onsite home evaluation with the client and the client's daughter. During the home evaluation, it is obvious that the hallway to the bathroom is too narrow for the client to turn the wheelchair without assistance. What would be the MOST APPROPRIATE transitional recommendation to include in the report? A. Referral to homecare occupational therapy for wheelchair mobility training B. Training in a 3-point wheelchair-turning technique in a narrow hallway C. Wearing an adult diaper and refraining from using the bathroom D. Securely placing a bedside commode between the bed and the side wall

A

A _____________________ is part of a vocational program that are generally time limited (3-6 months) with discharge to competitive employment, supportive employment, or rehabilitation workshops. A) Transitional employment programs (TEPs) B) Rehabilitation workshops C) Employee Assistance Programs

C

A _____________________ is part of a vocational program that provides ongoing support, intervention, and referrals as needed to a company's employees to enable these individuals to maintain this employment. A) Transitional Employment programs (TEPs) B) Rehabilitation Worships C) Employee Assistance Programs

B

A _______________________ is part of a vocational program where discharge is not always a goal. Mainatenance of the person in these structured work environments can be desired objective for some individuals while others will be discharged to other programs to to work. A) Transitional employment programs (TEPs) B) Rehabilitation workshops (sheltered workshops) C) Employee Assistance Programs

B

A client did not attend the occupational therapy session because of an illness. According to the Guidelines for Documentation of Occupational Therapy, what is the BEST type of documentation to note nonattendance in a timely manner? A.Reassessment report B. Service contact C. Plan of care D. Monthly progress report

B

A client had surgery 6 weeks ago. The surgeon used a plate to fix a metacarpal shaft fracture. The client is experiencing moderate edema that makes it difficult to make a fist. Grasping containers and manipulating clothing fasteners are the most difficult tasks. What is the BEST approach for the OTR® to take to address the client's edema? A. Pneumatic pump and cold packs B. Cold packs and compression glove C. Deep retrograde massage and elevation D. A bulky dressing for the arm

B

A client has a diagnosis of Brown-Sequard syndrome. Which of the following can the therapist expect as a result of this injury? A. Sensory loss on the contralateral side, pain and temperature (hypalgesia) on the ipsilateral side. B. Sensory loss on the ipsilateral side, pain and temperature (hypalgesia) on the contralateral side. C. Sensory loss, pain and temperature (hypalgesia) loss on the ipsilateral side. D. Sensory loss, pain and temperature (hypalgesia) loss on the contralateral side.

A

A client has a unilateral transradial amputation. The client will be fitted with a prosthetic device in 1 month. One of the client's goals is to be independent with BADL as soon as possible. The client refuses to use the uninvolved hand for toileting tasks, citing cultural reasons. What INITIAL action should the OTR® take in this situation? A. Talk with the client about using alternative strategies. B. Have the client practice the task using assistive devices. C. Revise the client's long-term goals based on cultural limitations. D. Focus on other tasks that are not impacted by cultural restrictions.

C

A client has been in a SNF for the past couple of weeks following a left CVA. The client has met all of the goals, and the team is about to discharge to home. Which assessment would be the most appropriate to determine the patient's inititiation, organization, safety, and task completion? A. Allen Cognitive Level Test B. Assessment of Motor Process Skills (AMPS) C. Executive Function Performance Test (EFPT) D. Rivermead Behavioral Memory Test

A

A client in an acute psychiatric hospital following severe depression due to the death of a spouse is being discharged home. The client states that he/she does not know what to do once he/she is discharged home. The client has expressed interest in finding new hobbies to help occupy his/her time. The OT would like to know the client's knowledge of leisure opportunities, environmental resources and barriers, and leisure characteristics that are motivating and interesting to the person. Which assessment would be most appropriate for the OT to administer to this client? A. Leisure Diagnostic Battery (LBD) B. Play History C. Meaningfulness of Activity Scale D. Minnesota Leisure time Physical Activity Questionnaire

C (always address pain first)

A client is recovering from an exacerbation of rheumatoid arthritis and is participating in outpatient occupational therapy. The initial evaluation indicates decreased active ROM of bilateral wrists and hands, bilateral ulnar drift, moderate edema, and a pain rating of 7 out of 10 on the visual analog scale. The client currently depends on the spouse to assist with all ADL, but has a goal to increase independence with self-care tasks. Which of the following statements would be BEST to include as a short-term goal in the client's intervention plan? A. Control MCP joint ulnar deviation by the end of the second session using bilateral hand-based splints during self-feeding. B. Increase active ROM of the digits to within functional limits for independence with grooming by the sixth occupational therapy session. C. Subjective report of pain during dressing tasks will decrease by 2 levels on a visual analog scale within 2 weeks. D. Verbalize daily use of joint protection techniques during oral hygiene tasks within 2 weeks.

A

A client left employment as an elementary school teacher because of impairments from a stroke 12 months previously and is now interested in pursuing a new occupation. A work skills interest assessment and performance skill measures indicate suitability for work as an administrative assistant. The client needs assistance in entering this new occupation and working with impairments. Which program would BEST provide this service? A.Vocational rehabilitation B.Supported employment C.Community center D.Employer consultation

A

Prone Position: At which age does a child shift weight on forearms and reaches forward. Bears weight and shifts weight on extended arms, legs are closer together and thighs roll inward toward natural alignment. Hips are flat on surface? A) 5-6 months B) 5-8 months C) 0-2 months D) 0 months

B (Can be simulated. Living skills include: basic communication, personal care and hygiene, med management, personal healthcare, time awareness, eating, dressing, safety, use of phone, transportation, maintenance of clothing, use of money)

A client living in a group home is in the process of transitioning to a half way house. The OT working with the client wants to assess a client's actual performance of basic living skills needed to function in his/her environment. The therapist assesses the client during his/her morning routine including medication management, getting on and off the bus, washing his/her clothes, and during a supervised cooking task. Which evaluation is the therapist using to assess these areas of ADLs/IADLs? A. Test of Grocery Shopping B. Milwaukee Evaluation of Daily Living Skills (MEDLS) C. Assessment of Motor and Process Skills (AMPS) D. Kohlman Evaluation of Living Skills

D

A client recovering from a CVA has left sided weakness and dysphagia. Using a direct intervention for oral motor control to successfully swallow ingested food would entail A) providing pureed, thick liquids B) providing thermal stimulation to the inferior faucial arches C) Tilting the person's head back and toward the left side D) providing small, warm boluses.

A (Oculomotor function is responsible for the symptoms the client is reporting. When images double side by side for near-vision tasks, pupils are asymmetrical, and ptosis (droopiness) of the eyelid occur, cranial nerves may be injured.)

A client reports diplopia, or side-by-side double images, and demonstrates asymmetrical pupil sizes and a droopy eyelid. What would be MOST appropriate for the OTR® to evaluate? A. Oculomotor function B. Visual acuity deficit C. Contrast sensitivity D. Visual field deficit

A

A client reports visual stress, fatigue, and frequent double vision. On evaluation, the client demonstrates asymmetrical eye movement and difficulty moving the eyes together. The client would like to return to continuous reading tasks. Which intervention would be MOST appropriate for the OTR® to recommend? A. Occlusion under the supervision of a physician B. Eye exercises to improve eye coordination C. Additional task lighting when reading D. Visual search and scanning tasks with mobility

A (Kyphosis often leads to deficits in eating skills (e.g., the ability to keep and manipulate food or fluid in the mouth and swallow it)

A client who had a total hip replacement 2 days ago with severe kyphosis as a result of osteoporosis is referred to occupational therapy. The chart notes that the client has been demonstrating signs of malnutrition for the past 4 months. What is the FIRST evaluation the OTR® should complete? A. Evaluation of eating B. Evaluation of meal preparation C. Evaluation of knowledge of nutrition D. Evaluation of feeding

A (the six areas assessed are dressing, shopping, making toast, making a phone call, washing, and traveling)

A client who has severe depression is performing six functional ADL tasks that require cognitive processing skills based on Allen's Cognitive Disabilities Model. The therapist provides demonstration, reassurance, cueing, and more directions (if needed) to facilitate task performance. Which assessment is the client performing? A. Cognitive Performance Test B. Mini Mental Exam C. Kohlman Evaluation of Living Skills (KELS) D. Klein-Bell Activities of Daily Living Scale (K-B Scale)

C

A client who suffered a stroke would like to return to work as a computer specialist. The OTR® is asked to explain vocational evaluation to the client. What explanation should the OTR® provide? A. Vocational evaluation determines the essential functions of a job area to conduct work tolerance screenings. B. Vocational evaluation identifies safety risks that may impede a worker's ability to complete job tasks. C. Vocational evaluation assesses a client's readiness and ability to engage in a particular occupation. D. Vocational evaluation determines an organization's compliance with the Americans With Disabilities Act.

C (Vocational evaluation may be conducted for a person who has not worked previously, who has been injured and is preparing to return to work, or is unable to return to a previous job. Vocational evaluation may be general or specific addressing a person's potential for work or readiness to return to a specific occupation.)

A client who suffered a stroke would like to return to work as a computer specialist. The OTR® is asked to explain vocational evaluation to the client. What explanation should the OTR® provide? A. Vocational evaluation determines the essential functions of a job area to conduct work tolerance screenings. B. Vocational evaluation identifies safety risks that may impede a worker's ability to complete job tasks. C. Vocational evaluation assesses a client's readiness and ability to engage in a particular occupation. D. Vocational evaluation determines an organization's compliance with the Americans With Disabilities Act.

B (A client with a C6 SCI may not be able to grasp a buttonhook with the fingers, so a palmar cuff would provide the needed grasp.)

A client with a C6 spinal cord injury (SCI) is performing a dressing task. Which type of equipment would the OTR® most likely give the client to assist with buttoning a shirt? A. No adaptive equipment would be needed B. Palmar-cuff buttonhook C. Extended buttonhook D. Standard buttonhook

A (Opposition and palmar abduction represent the functional position of the thumb that best allows pinch.)

A client with a chronic median nerve compression at the carpal tunnel has severely diminished functional pinch. In what position should the thumb be splinted to facilitate functional pinch? A. The thumb should be splinted in opposition and palmar abduction to facilitate thumb-to-tip prehension. B. The thumb should not be included in a carpal tunnel orthotic to allow for functional prehension. C. The thumb should be splinted in opposition and radial abduction to facilitate thumb-to-tip prehension. D. The thumb should be splinted in extension to limit shortening of the extensor pollicus longus.

D

A client with a learning disability is interested in exploring work opportunities after high school graduation. The client has difficulty sequencing more than three-step directions and lacks attention to detail. Which option would be considered a component of the student's transitional program plan? A. Completion of a transition-related evaluation by a vocational rehabilitation counselor to identify the student's strengths, weaknesses, and interests B. Completion of a job site analysis with physical demands to determine the feasibility of the client's performing specific tasks C. Provision of written directions to complete a three-step task and use of samples to promote problem solving D. Situational observation, interview, and activity analysis to determine a match between the client's abilities and expected performance

B (Bending forward at the kitchen sink does not require as much hip flexion as bending forward at a typical bathroom sink; many clients are able to wash hair at the kitchen sink without compromising hip precautions.)

A client with recent hip replacement surgery is concerned about how to shampoo hair while not able to use the shower at home. What recommendation from the OTR® will most appropriately address the client's concern? A. The client may shampoo the hair while standing in the shower, covering the hip with a plastic bag. B. The client may shampoo the hair while sitting on a stool or standing at the kitchen sink. C. The client may shampoo the hair while in a wheelchair at the bathroom sink. D. The client may shampoo the hair while standing at the bathroom sink.

A (The client demonstrates generalization, which is the ability to take a strategy used with one task and apply that strategy to a new task.) (B: Transfer occurs when clients dress themselves at home in the same way they did in their hospital room. C: Automaticity is the ability to perform tasks with little or no contribution of consciousness; this client has not yet demonstrated automaticity. D: Acquisition is a training strategy that relies on conscious control and requires practice and drill exercises. Acquisition is a cognitive strategy, not a method of learning.)

A client with stroke is taught to dress the weaker side first when donning a button-down shirt. The client then initiates putting the weaker lower extremity into the pant leg. What learning has occurred for the client? A. Generalization B. Transfer C. Automaticity D. Acquisition

B (determines individual's capacity for lifting, carrying, climbing, industrial pulling and pushing, balance while walking, motor coordination, standing, whole body ROM, and finger and hand dexterity)

A construction worker is being discharged from outpatient therapy following chronic back pain secondary to a herniated disc. Workman's comp requires that the client be evaluated for appropriateness to return to work. Which of the following assessments is MOST appropriate for this client? A. Jacob's Prevocational Assessment (JPVA) B. Epic Functional Evaluation System C. Testing Orientation, and Work Evaluation in Rehabilitation (TOWER) D. Worker Role Interview

C

A group of adults in a psychiatric day treatment program work together to implement a long-range activity which requires them to work as a team to complete. The focus of this group is to enable members to identify and meet the needs of themselves and others (e.g. safety, esteem). Which group model does this follow? A. Project B. Mature C. Ego-centric Cooperative D. Cooperative

B

A group of researchers audit medical records after interventions were rendered to ensure appropriate care was given. Which of the following reviews is being conducted? A) Concurrent review B) Retrospective review C) Prospective review D) Utilization review

A

A group that is designed as a continuum of groups consisting of parallel, project, egocentric-cooperative, cooperative, and mature groups. Type of group members are individuals with decreased group interaction skills. Which activity group does this follow? A. Developmental B. Task-oriented group C. Topical group D. Thematic group

A

A group that is designed to assist members in acquiring the knowledge, skills, and/or attitudes needed to perform a specific activity. Members of this group must have minimal interaction skill level equal to a parallel group skill level. Which activity group does this follow? A. Evaluation group B. Thematic group C. Topical group D. Task-oriented group

B

A group that is designed to discuss specific activities that members are engaged in outside of the group to enable them to engage in the activities in a more effective, need-satisfying manner. The members of this group are individuals who share similar current or anticipatory problems in functioning. Members must be at an egocentric-cooperative group level with sufficient verbal and cognitive skills to engage in discussion and problem solve. Suitable activities include verbal discussion on a circumscribed activity that members are engaged in or will be engaged in outside of group. Which activity group does this follow? A. Developmental group B. Topical group C. Task-oriented group D. Evaluation group

C

A group that is designed to enable the client and the therapist to assess client's skills, assets, and limitations regarding group interaction. Members of this group are individuals who lack group interaction skills. The therapist orients clients to group's purpose, selects activities that require collaboration and provides needed supplies, and does not participate or intervene in group, but observes and reports members' interaction and functional skill level to the treatment team. Suitable activities include tasks that require interaction to complete and can be completed in one session. Which activity group does this follow? A. Thematic group B. Developmental group C. Instrumental group D. Task-oriented group

B

A group that is designed to help members function at their highest possible level for as long as possible. Members of this group are individuals who have demonstrated in treatment an inability to change or progress; and who cannot independently meet their mental health needs and/or need assistance to maintain function due to cognitive psychosocial, perceptual-motor, and/or social deficits. Suitable activities include successfully completing activities with structure and assistance of therapist as needed, meet mental health needs, maintain function by providing sensory, cognitive, perceptual-motor, and social input. A. Task-oriented group B. Instrumental group C. Developmental group D. Evaluation group

B

A group that is designed to increase clients' awareness of their needs, values, ideas, feelings, and behaviors as the engage in a group task and improve intra- and interpsychic functioning by focusing on problems which emerge in the process of choosing, planning and implementing a group activity. Members of this group include individuals whose primary dysfunction is in the coginitive and socioemotional areas due to psychological or physical trauma and clients with fair verbal skills who can interact with others. Suitable activities are chosen by members and will create an end product or demonstrable service for the group itself or for persons outside the group. Activities are selected, planned, and carried out by members with the understanding that the task is a means to study, understand, and practice behavior. Which activity group does this follow? A. Evaluation group B. Task-oriented group C. Topical group D. Thematic group

C

An OT working in the schools is working with a group of high schoolers who are unclear about their vocational interests. Which assessment would be most appropriate for the OT to help them determine a list of interest-compatible college majors? A. Jacob's Prevocational Assessment (JPVA) B. McCarron-Dial System C. Vocational Interest Inventory - Revised (VII-R) D. Epic Functional Evaluation System

A

A high school student who is developmentally delayed is interested in finding an afterschool job. Which assessment provides a therapist working with this individual observed areas of difficulty during task performance, time for task completion, and comments on behavior during the task: A. Jacob's Prevocational Assessment (JPVA) B. McCarron-Dial System C. Valpar Component Work Sample D. Testing Orientation, and Work Evaluation in Rehabilitation (TOWER)

C

A high school student with CP participates in a prevocational group at school. To help the therapist working with the student obtain a better understanding of the student's skills, she administers an assessment that includes five main areas: cognitive, sensory, motor, emotional, and coping/adaptive behaviors. Which assessment is the OT administering to the student? A. Worker Role Interview B. Epic Functional Evaluation System C. McCarron-Dial System D. Valpar Component Work Sample

A

A hospital is evaluating ongoing intervention programs during various patient's hospitalization, outpatient, or home care treatment to ensure appropriate care is being delivered. This is part of the their quality improvement (QI) program that seeks to continually increase quality. Which of the following reviews is being conducted? A) Concurrent review B) Retrospective review C) Prospective review D) Utilization review

B

A lack of energy and initiative, which is often misinterpreted as lack of motivation is A) Anhedonia B) Anergia C) Affective flattening D) Alogia

True

A limited data set that does not include any identifiable information can be used in research without patient approval (e.g. diagnosis, age, length of stay).

6 weeks

A mallet finger is the avulsion of the terminal tendon and is splinted in full extension for ______ _______________.

B (Injury prevention programs aim to control work-related musculoskeletal disorders by implementing processes focused on identifying risk for musculoskeletal problems, providing skill training for management and worker evaluation of risk factors, and developing effective controls for identified risk factors.)

A manager at a clothing manufacturing plant hires an OTR® as a consultant to improve injury management in the company. Which goal would be the MOST reasonable for this program? A. The program will reduce injuries related to work-related musculoskeletal disorders. B. The organization will control the severity and incidence of work-related musculoskeletal disorders. C. Management will drive the program by making decisions on wellness activities to be implemented. D. Workers will improve performance, becoming more efficient and increasing productivity.

D

A manager is reviewing resources within a facility to determine medical necessity and cost efficiency through reimbursement claims to determine the most efficient and cost-effective care. Which of the following reviews is the manager conducting? A) Concurrent review B) Retrospective review C) Prospective review D) Utilization review

Proximal fracture

A metacarpal fracture is known as a _______________ ______________.

C

A parent notices that her child has difficulty getting up from the floor to a standing position. He often uses his hands to crawl up his thighs to get to standing (known as Gower's sign). One he is up and walking she notices that he presents with a trendelenurg (waddling) gait and falls frequently. What diagnosis may this child be presenting with? A) Cerebral Palsy B) Progressive Supranuclear Palsy C) Duchenne's Muscular Dystrophy D) spina bifida with a myelomeningocele

Stage II

A patient diagnosed with Parkinson's displays bilateral tremor, rigidity, akinesia, with or without axial signs, independent with ADLs, no balance impairment. What stage of the disease is this person most likely experiencing?

Stage I

A patient diagnosed with Parkinson's displays unilateral tremor, rigidity, akinesia, and minimal or no functional impairment. What stage of the disease is this person most likely experiencing?

Stage III

A patient diagnosed with Parkinson's displays worsening symptoms, first signs of impaired righting reflexes, onset of disability in ADL performance, can lead ind. life. What stage of the disease is this person most likely experiencing?

Stage V

A patient diagnosed with Parkinson's is confined to a wheelchair or bed and requires maximal assist. What stage of the disease is this person most likely experiencing?

Stage IV

A patient diagnosed with Parkinson's requires help with some or all ADLs, unable to live alone without some assistance, able to walk and stand unaided. What stage of the disease is this person most likely experiencing?

A

A patient has been admitted to a SNF after an acute care stay following a TBI. The OT working with the patient wants to know the patient's basic cognitive functions (orientation, visual, spatial perception, visual motor organization, and thinking operations). Which of the following tests would be MOST appropriate? A. Lowenstein OT Cognitive Assessment (LOTCA) B. Executive Function Performance Test (EFPT) C. Rivermead Perceptual Assessment Battery D. Montreal Cognitive Assessment (MOCA)

GLASGOW Coma Scale

A patient has sustained a head injury less than 3 hours ago, what is the BEST assessment to use first to determine the patient's level of consciousness?

Left

A patient in a SNF demonstrates difficulties with verbal memory, bilateral auditory reception, speech, and processing of verbal auditory information. Which side of his brain was likely affected by a CVA?

B

A patient in a SNF has a C4 spinal cord injury. What can an OTR expect the patient to do? A. Chew and swallow only B. Chew, swallow, and drink from a glass with a long straw. C. Self feed using a suspension sling or a mobile arm support D. Independent with all ADLs

B (Used to assess pre-morbid physical activity levels, relationship with general health, disease, cardiovascular fitness, and weight control.)

A patient in a sub-acute care hospital is status post CABG due to a history of heart disease. He/she works in a high stress environment and has expressed interest in creating goals to promote a healthier lifestyle and begin engaging in more physical activity. Which assessment would be best to administer to this client? A. Activity Index B. Minnesota Leisure time Physical Activity Questionnaire C. Leisure Diagnostic Battery D. Interest Checklist

D

A patient in the acute care unit of the hospital presents with limitations in orientation, attention and concentration, and orientation. The OT working with this patient is concerned the patient may have mild cognitive dysfunction. Which assessment would be most appropriate to use with this patient? A. Executive Function Performance Test (EFPT) B. Behavioral Inattention Test C. Rivermead Perceptual Assessment Battery D. Montreal Cognitive Assessment (MOCA)

Right

A patient is diagnosed with a CVA, he demonstrates difficulty with visual spatial processing, processing of nonverbal auditory information, attention to incoming stimuli, and interpretation of tonal inflections. Which side of the brain was likely affected?

D

A patient is diagnosed with a conus medullaris SCI. What is the most likely outcome for this diagnosis? A. Muscle strength is preserved, loss of proprioception + vibration sensation + loss of two point discrimination +loss of light touch B. Sensory loss on the ipsilateral side, pain and temperature (hypalgesia) on the contralateral side. C. Complete loss of muscle strength below the level of injury, pain and temperature loss, proprioception and touch preserved. D. Lower extremity motor and sensory loss and an areflexic bowel and bladder.

B

A patient presents with difficulty with new learning and current events. The family is worried that the patient may have dementia. Which assessment would be most appropriate to assess cognitive impairment associated with dementia? A. A-ONE (Arnadottir OT Neurobehavioral Evaluation) B. Mini-Mental State Examination C. Allen Cognitive Level Test D. Rivermead Behavioral Memory Test

C (Includes 9 activity-based subtests - picture scanning, menu reading, map navigation, address and sentence copying, card sorting, article reading, telephone dialing, coin sorting, and telling/setting time: 6 pen/paper subtests - line crossing, star cancellation, letter cancellation, figure and shape copying, and line bisection)

A patient presents with unilateral neglect. An OT would like to examine the presence of neglect and its impact on the client's performance on functional tasks. Which of the following assessments would she be MOST likely to use? A. AMPS (Assessment of Motor and Process Skills) B. Cognistat Neurobehavioral Cognitive Status Exam C. Behavioral Inattention Test D. Lowenstein OT Cognitive Assessment (LOTCA)

Severe (<8)

A patient sustained a head injury and scored a 2 on best eye response (on eyes opening in response to pain), a 3 on verbal response (inappropriate words, random or exclamatory speech, but not conversational exchange), and a 2 for best motor response (extension to pain, decerebrate response). His total score was a 7. Would this score be interpreted as severe, moderate, or minor on the GLASGOW Scale?

Moderate (9-12)

A patient sustained a head injury and scored a 3 on best eye response (eyes opening in response to speech), a 4 on verbal response (confused, patient responds to questions coherently but there is some disorientation and confusion), and a 5 for best motor response (Localizes to pain, purposeful movements towards changing painful stimuli). His total score was a 12. Would this score be interpreted as severe, moderate, or minor on the GLASGOW Scale?

Minor (13-15)

A patient sustained a head injury and scored a 4 on best eye response (on eyes opening spontaneously), a 5 on verbal response (oriented, responds coherently and appropriately to questions), and a 5 for best motor response (extension to pain, decerebrate response). His total score was a 14. Would this score be interpreted as severe, moderate, or minor on the GLASGOW Scale?

A

A patient who sustained a right CVA and presents with cognitive and perceptual impairments. The therapist working with him wants to assess his BADL and IADL performance. Which of the following assessments is MOST appropriate to use for this patient? A. Assessment of Motor and Process Skills (AMPS) B. Allen Cognitive Level Test C. EFPT (Executive function performance test D. Mini-Mental State Examination

C

A patient with a C5 spinal cord injury lives at home with family. The patient's spouse reports feeling tired and frustrated as the patient is not able to do all of their self cares. The patient can dress the upper body and bathe with minimal to moderate assistance. However, the patient is completely dependent for feeding and grooming. Which of the following would be the best recommendation for the OTR to make? A. Provide information on a universal cuff or a tenodesis splint. B. Refer the spouse to a SCI family support group. C. Provide information on a suspension sling or a mobile arm support. D. Respond to the spouse's frustration by saying, "I hear what you are saying, that must be really difficult for you."

A

A patient with a C6 spinal cord injury lives at home with family. The patient's spouse reports feeling tired and frustrated as the patient is not able to do all of their self cares. The patient can dress the upper body and bathe with minimal to moderate assistance. However, the patient is completely dependent for feeding and grooming. Which of the following would be the best recommendation for the OTR to make? A. Provide information on a universal cuff or a tenodesis splint. B. Refer the spouse to a SCI family support group. C. Provide information on a suspension sling or a mobile arm support. D. Respond to the spouse's frustration by saying, "I hear what you are saying, that must be really difficult for you."

A (Agnosia can also be the inablitiy to understand and interpret the significance of sensory input)

A patient's family expresses concern that their family member does not recognize them or other familiar objects. What is the most likely experiencing? A) Agnosia B) Adiadochokinesia C) Receptive aphasia D) Depersonalization

B (Splinting = up to 6 weeks, PROM = 8 weeks, strengthening = 10 weeks)

A person incurred a rupture of the ulnar collateral ligament of the MCP joint of the thumb 7 weeks ago. Which intervention approach should to OT use with this client? A. Splinting B. AROM C. PROM D. Strengthening.

C

A person with Level 1 autism spectrum disorder without intellectual impairment (Asperger syndrome) is having difficulty in the transition to a new adult foster care home, and an OTR® is contracted to make four home visits to assess the client and provide appropriate intervention. When the OTR® arrives for the first visit and attempts to interview the client, the client angrily refuses to speak with the OTR®. Which option is BEST in response? A. Stay long enough to convince the client to cooperate so that the placement can be successful B. Engage the client in a favorite game and convince the client to work together C. Leave the home, allowing the client to refuse occupational therapy services D. Tell the client that the OTR® will stop hassling the client after four visits

C

A resident in a long term care facility has a severe elbow contracture. The OTR® has just finished applying a bivalved splint to provide low load prolonged stress to the affected elbow. What MUST the OTR® do as part of the care plan for maximizing the effectiveness of this splinting intervention? A. Supplement the splint wear schedule with high brief-load stretches for 1-2 minutes at least 3 times per day. B. Advise the resident that the stretch will be uncomfortable but needs to be tolerated for at least several hours at a time. C. Provide nursing staff with specific instructions for a graded daily splint-wearing schedule. D. Ensure the evening nursing shift has documented orders to apply the splint just before the resident goes to sleep.

D

A retired school teacher had triple bypass surgery and is being discharged home from a SNF. He/she states that his/her day is so structured at the facility, he/she is afraid to go home and have nothing to do. The occupational therapist would like to analyze the patient's level of enjoyment, motivational source, perception of competence, and participation in leisure in order to promote adaptive leisure functioning. Which assessment would be the best to administer to this client? A. Leisure Satisfaction Questionnaire B. Activity Index C. Interest Checklist D. Meaningfulness of Activity Scale

A (Once the evaluation is completed and the frequency of visits is set, the school district must provide the services written in the student's individualized education plan to maintain compliance with the Individuals with Disabilities Education Improvement Act of 2004.)

A school district's supervising OTR® receives a phone call from an irate parent of a middle school student whose individualized education plan (IEP) includes occupational therapy twice a week. The student did not receive occupational therapy for 3 weeks because one of the district's OTR®s was out with a work-related injury. What is the OTR®'s BEST option in response? A. Ensure that occupational therapy services are provided to this child, because it is included in the child's IEP B. Put the child on a waiting list while the therapist is out C. Reduce the frequency of occupational therapy visits to once a week while the therapist is out D. Discharge the child from occupational therapy because the parent is difficult to deal with

B (The student's response to occupational therapy intervention and teacher data is important in implementing an effective intervention plan. The intervention plan appears to need modification on the basis of the student's response.)

A student in the third grade with a learning disability has attended school-based occupational therapy for several years to improve visual-perceptual skills for completing curriculum-based school work. The latest update report to the parents indicates the student is not making progress toward the stated Individualized Education Program (IEP) goals. This report is similar to the previous two reports. What action should the OTR® take NEXT to address the lack of progress? A. Continue occupational therapy intervention knowing that developmental progress due to maturity is still possible. B. Discuss alternative classroom modifications and adaptations with the student's teacher. C. Request the special education team schedule an interim meeting to modify the student's IEP. D. Send a letter to the student's parents informing them that occupational therapy is no longer beneficial to their child.

B (Current rehabilitation focus is on the place-and-train supportive employment approach, and evidence supports its use.) (A, C, D: Prevocational training, transitional employment, and sheltered workshops use the classic train-then-place methodology. Because of the emphasis on preparatory skills in these settings, opportunities to transfer skills to actual work settings yield a lower percentage of employment.)

A team of OTR®s begins to recognize the need for a program to promote work participation for the young adults with mental illness they serve. Which statement MOST accurately reflects the evidence about developing such a program? A. Prevocational training sufficiently prepares clients with mental illness for work. B. Supported employment programs with a "place-and-train" perspective are more effective than other vocational approaches. C. Transitional employment placement through a clubhouse model is a time-honored method for increasing job placement. D. Sheltered workshops are useful programs for advancing work participation.

D

A therapist acts as an advisor for a maternity group that engages in free expression of ideas and feelings. The focus of this group is to enable members to identify and meet socio-emotional needs. Which group model does this follow? A. Project B. Mature C. Ego-centric Cooperative D. Cooperative

B (the six areas assessed are bathing, dressing, toileting, transferring, continence, and feeding. The therapist can also interview the individual on performance)

A therapist in a skilled nursing facility is conducting a re-evaluation of a client who has a diagnosis of Parkinson's disease in order to assess his/her level of independent functioning and type of assistance required in six areas of ADL. The therapist observes activity performance and rates each activity as independent, some assistance required, or dependent. The scores are then converted into a letter score A-G. Which assessment is the therapist using? A. Cognitive Performance Test B. Katz Index of ADL C. Klein-Bell Activities of Daily Living Scale (K-B Scale) D. Barthel Index

Kinsesthesia (movement sense)

A therapist moves a segment and the client responds up or down. Is the therapist testing proprioception or kinesthesia?

Proprioception

A therapist positions the involved extremity and the client duplicates the position with the contralateral extremity. Is the therapist testing proprioception or kinesthesia?

Sensory

Cranial nerves I, II, and VIII are pure _______________: they carry special senses of smell, hearing, and equilibruim.

A

A therapist takes a client who sustained a TBI 10 months ago to his/her community grocery store. She gives him/her a list of 10 of specific items of specific sizes and asks him/her to locate and select the items at the lowest price. The therapist observes the client's performance of this task. Which assessment is the therapist using? A. Test of Grocery Shipping Skills B. Barthel Index C. Scoreable Self-Care Evaluation D. Routine Task Inventory (RTI)

D

A therapist working in a SNF is conducting an initial evaluation for a newly arrived individual who has sustained a right sided CVA. The therapist has completed a training course on rater calibration is administering and ADL/IADL assessment for the client with 3-5 tasks that the client has chosen as relevant and sufficiently challenging to obtain more information on the client's efficiency and safety during functional daily tasks. Which assessment would this therapist be MOST likely to use? A. Barthel Index B. Katz Index C. Milwaukee Evaluation of Daily Living Skills (MEDLS) D. Assessment of Motor and Process Skills (AMPS)

C (Information from this test is often used in research)

A therapist working in a day program is conducting an assessment of independent functioning in activities of daily living to a client who is developmentally delayed. The therapist observes and scores the individual's performance of 170 items in six areas. The OT uses the information to increase caregivers' understanding of the individual's need for assistance. Which assessment is the therapist using? A. Cognitive Performance Test B. Katz Index of ADL C. Klein-Bell Activities of Daily Living Scale (K-B Scale) D. Barthel Index

B

A therapist working in an assisted living is conducting an ADL assessment for an older adult who has recently returned from sub-acute hospitalization following hip surgery. The therapist reviews the client's medical chart, directly observes the client's morning routine performance, and conducts an interview with the client and the staff at the assisted living facility. Which assessment is this therapist performing? A. Katz Index B. Barthel Index C. FIM D. Scoreable Self-Care Evaluation

B

A two and a half year old male has recently been diagnosed with muscular dystrophy. What is the BEST explanation for an OT to give the family regarding the progression of this diagnosis? A) Progression of muscular dystrophy is steady throughout the lifespan. B) Progression of muscular dystrophy is often rapid and most children require a wheelchair by age 9. C) Most children with muscular dystrophy live into late adulthood. D) It's best to talk to a physician about concerns regarding this diagnosis as they can give you details on what to expect.

Edema

A volumeter or centimeter tape measures ________________.

C

A worker is injured at a construction site. The worker's case manager requests a referral to an OTR® through the employer's workers' compensation carrier. The employer frequently checks in with the worker and the case manager about the status of the case. Who is the client in this situation? A. Insurance carrier B. Case manager C. Worker D. Employer

independent

AOTA supports the i___________________ practice of OTAs with advanced level skills who work for independent living centers.

Avulsion injuries

A_______________ i________________Occur with the tendon separates from the bone and its insertion and removes the bone material with the tendon.

24 months

Able to chew most meats and raw veggies

Gross Body Movements

Allen cognitive level 2 is characterized by G___________ B____________ M_______________

Manual Actions

Allen cognitive level 3 is characterized by M_______________ A__________________

Familiar Activity

Allen cognitive level 4 is characterized by F______________ A_______________.

Learning New Activity

Allen cognitive level 5 is characterized by L_______________ N________ A_________________.

Planning New Activity

Allen cognitive level 6 is characterized by P______________ N_________ A_________________.

C (Serves as a technical expert to recommend commercial products, adapt available devices, and/or modify existing devices.)

An OT and speech pathologist are working together with a patient and his/her family who are interested in an augmentative device. The client has a tilt in space wheelchair and is able to move his/her shoulder to 30 degrees flexion and elbow flexion is 20 degrees. The OT and SLP would like to implement a device that is activated by the client's head. Which member of the interdisciplinary team should the OT and speech pathologist consult? A) Audiologist B) Certified prosthetist C) Biomedical Engineer D) Physician

C (primary intervention)

An OT consultant is developing a violence prevention program for a school system. Which of the following interventions would the OT likely implement? A. Consult with administration to identify which processes have already been put into place and build on it. B. Work with students who have been identified as at risk for violence and help them develop coping strategies for aggressive behaviors. C. Implement activities for staff and students related to conflict resolution and assertiveness training in order to prevent aggressive behaviors and promote effective coping skills.

A (Perform a simple cooking task)

An OT in a sub-acute care facility is preparing to discharge a patient who sustained a left sided CVA one month ago. The client is preparing to discharge home with his/her spouse. The therapist wants to measure the client's judgment, planning, and organizational skills. Which assessment would be most beneficial to obtain this information? A. Kitchen Task Assessment (KTA) B. Klein-Bell Activities of Daily Living Scale (K-B Scale) C. Scoreable Self-Care Evaluation D. Test of Grocery Shopping Skills (TOGSS)

A (Vomiting, seizures, and deterioration of physical and/or cognitive functioning are also signs associated with an infected shunt.)

An OT in the schools is working with a 5 year old diagnosed with spina bifida with a myelomeningocele. The child presents to OT with lethargy, fever, and headache. What is the BEST option for the OT to do in this situation? A) Call the child's primary physician and report that the OT believes the child may have an infected shunt. B) Call the child's caregiver to come and pick her up. C) Continue with OT but at a slower pace, this is a common occurrence for children with spina bifida with myelomeningocele. D) Send the child to the nurse to administer medication for these symptoms.

A

An OT provides an adolescent in a school based setting a checklist containing 80 items and instructs the client to check their level of interest in each activity. The assessment is used to measure level of interest in the list of 80 leisure activities and gain perspective on how leisure interests and involvement has evolved over time. Which Assessment is the OT using? A. Interest Checklist B. Play History C. Meaningfulness of Activity Scale D. Leisure Diagnostic Battery (LBD)

D (17 basic living skills needed to live independently in 5 main areas: self-care, safetly and health, money management, transportation and telephone, work and leisure.)

An OT working in an acute psychiatric hospital with a 17 year old whose goal is to eventually live independently. The therapist wants to assess how the client's functional level for ADLs/IADLs. Which assessment would be MOST appropriate to use? A. KATZ Index of ADL B. Kitchen Task Assessment (KTA) C. Routine Task Inventory D. Kohlman Evaluation of Living Skills

C

An OT working in an assisted living is assessing an individual's perception of leisure and the extent to which he/she participates in leisure activities. Which assessment provides information about the client's level of participation in leisure activities and additional activities of interest? A. Leisure Diagnostic Battery (LDB) B. Leisure Satisfaction Questionnaire C. Activity Index D. Minnesota Leisure time Physical Activity Questionnaire

D (ADLs: grooming, dressing,, bathing, walking, feeding, and toileting. IADLs: housekeeping, preparing food, spending money, taking meds, doing laundry, telephoning, and traveling.)

An OT working in home health is working with a client who has a cognitive impairment. The OT wants to assess the client's level of impairment in activities of daily living according to Allen's Model of Cognitive Disabilities. The assessment contains 6 physical scales in areas of ADLs and 8 instrumental scales in IADLs. The therapist chooses to observe the client in each of the scales rather than use a self-report or a caregiver report. Which assessment is the therapist using? A. Cognitive Performance Test B. Katz Index of ADL C. FIM D. Routine Task Inventory (RTI)

A

An OT working in pediatrics can expect a child to achieve a Digital pronate grasp: writing tool held with fingers, wrist neutral with light ulnar deviation, and forearm pronated; arm moves as a unit? A. 2-3 years B. 4 years C. 1-1.5 years D. 9 months

C

An OT working in pediatrics can expect a child to achieve a dynamic tripod posture: writing tool held with cprecise opposition of distal phalanges of thumb, index, and middle fingers, ring and little fingers flexed to form stable arch, wrist slightly extended, grasped distally, MCP joints stabilized during fine, localized movements of PIP joints? g A. 6+ years B. 3 years C. 4.5-6 years D. 3.5-4 years

C

An OT working in pediatrics can expect a child to achieve a palmar-supinate grasp: writing tool held with fisted hand, wrist slightly flexed and slightly supinated away from mid-postition; arm moves as a unit? A. 2-2.5 years B. 3 years C. 1-1.5 years D. 4.5-6 years

D

An OT working in pediatrics can expect a child to achieve a static tripod posture: writing tool held with crude approximation of thumb, index, and middle fingers, ring and little fingers only slightly flexed, grasped proximally with continual adjustments by other hand, no fine localized movements of digit components; hand moves as a unit? A. 2-2.5 years B. 3 years C. 4.5-6 years D. 3.5-4 years

B

An OT working in pediatrics can expect a child to pick up coins and move them to the palm of the hand (Finger-to-palm translation) by what age, according to Exner's Classification System? A. 2.5 years B. 12-15 months C. 18-24 months D. 5 months

B

An OT working in pediatrics can expect a child to pick up pennies with tumb and forefinger while storing them in the ulnar side of the same hand by what age, according to Exner's Classification System? A. 8-9 years B. 6-7 years C. 4-5 years D. 2-2.5 years

C

An OT working in pediatrics can expect a child to place coins in a slot (palm-to-finger translation) by what age, according to Exner's Classification System? A. 3-5 years B. 3-6+ years C. 2-2.5 years D. 6-7 years

B

An OT working in pediatrics can expect a child to roll a piece of clay into a ball (shift) by what age, according to Exner's Classification System? A. 6-7 years B. 3-6+ years C. 2-2.5 years D. 3-5 years

A

An OT working in pediatrics can expect a child to separate pieces of paper (Shift) by what age, according to Exner's Classification System? A. 3-5 years B. 2-2.5 years C. 4-6 years D. 12 months

A

An OT working in pediatrics can expect a child to shift on marker or pencil (shift) by what age, according to Exner's Classification System? A. 5-6 years B. 3-6+ years C. 2-2.5 years D. 3-5 years

A

An OT working in pediatrics can expect a child to turn a pencil over to erase (Complex rotation) by what age, according to Exner's Classification System? A. 6-7 years B. 3-6+ years C. 3-5 years D. 2-2.5 years

D

An OT working in pediatrics can expect a child to unscrew a small bottle cap (simple rotation) by what age, according to Exner's Classification System? A. 6-7 years B. 3-6+ years C. 3-5 years D. 2-2.5 years

C

An OT working in transitional housing is working with a veteran who has a diagnosis of post traumatic stress disorder. The therapist wants to assess the client's personal care, housekeeping chores, work and leisure, and financial management skills. The OT administers a motivational questionnaire to assess the veteran's values/beliefs about self-care skills and then administers each task. Which assessment is this therapist using? A. Assessment of Motor and Process Skills (AMPS) B. Barthel Index C. Scoreable Self-Care Evaluation D. Routine Task Inventory (RTI)

D

An OT working with a 10 month old can expect grasping of a pellet skills according to Erhardt Prehension Developmental Levels to be: A. Fine Pincer grasp between fingertips or fingernails, distal thumb joint flexed. B. Raking and contacting object C. Inferior-scissors grasp: raking object into palm with adducted totally flexed thumb and all felxed fingers, or two parially extended fingers D. Pincer grasp: between distal pads of thumb and index finger, distal thumb joint slightly flexed, thumb opposed.

B

An OT working with a 12 month old can expect grasping of a pellet skills according to Erhardt Prehension Developmental Levels to be: A. Scissors grasp: Between thumb and side of curled index finger, distal thumb joint slightly flexed, proximal thumb joint extended. B. Fine Pincer grasp between fingertips or fingernails, distal thumb joint flexed. C. inferior-scissors grasp: raking object into palm with adducted totally flexed thumb and all felxed fingers, or two parially extended fingers D. Pincer grasp: between distal pads of thumb and index finger, distal thumb joint slightly flexed, thumb opposed.

B

An OT working with a 3 month old can expect grasping of a pellet skills according to Erhardt Prehension Developmental Levels to be: A. Raking and contacting the object B. No attempt to grasp, but visually attends to object C. No voluntary grasp or visual attention to the object D. Scissors grasp

A

An OT working with a 3 month old can expect grasping of the cube skills according to Erhardt Prehension Developmental Levels to be: A. Visually attends to object and may swipe. Sustained voluntary grasp possible only upon contact, ulnar side used, no thumb involvement, wrists felxed. B. Progression toward palmar grasp. Infant's thumb begins to adduct with fingers pressed against ulnar side of palm. C. Visually attends to object, approaches if within 1 inch, contact results in hand pulling object back to squeeze precariously against the other hand or body. No thumb involvement. D. Palmar grasp: Fingers on top surface of object press it into center of palm with thumb adducted

C

An OT working with a 4 month old can expect grasping of the cube skills according to Erhardt Prehension Developmental Levels to be: A. Visually attends to object and may swipe. Sustained voluntary grasp possible only upon contact, ulnar side used, no thumb involvement, wrists felxed. B. Progression toward palmar grasp. Infant's thumb begins to adduct with fingers pressed against ulnar side of palm. C. Visually attends to object, approaches if within 1 inch, contact results in hand pulling object back to squeeze precariously against the other hand or body. No thumb involvement. D. Palmar grasp: Fingers on top surface of object press it into center of palm with thumb adducted

B

An OT working with a 4-5 month old can expect grasping of the cube skills according to Erhardt Prehension Developmental Levels to be: A. Visually attends to object and may swipe. Sustained voluntary grasp possible only upon contact, ulnar side used, no thumb involvement, wrists felxed. B. Progression toward palmar grasp. Infant's thumb begins to adduct with fingers pressed against ulnar side of palm. C. Visually attends to object, approaches if within 1 inch, contact results in hand pulling object back to squeeze precariously against the other hand or body. No thumb involvement. D. Palmar grasp: Fingers on top surface of object press it into center of palm with thumb adducted

D

An OT working with a 5 month old can expect grasping of the cube skills according to Erhardt Prehension Developmental Levels to be: A. Radial Palmar grasp: Fingers on far side of object press it against opposed thumb and radial side of palm. with wrist straight. B. Visually attends to object and may swipe. Sustained voluntary grasp possible only upon contact, ulnar side used, no thumb involvement, wrists flexed. C. Visually attends to object, approaches if within 1 inch, contact results in hand pulling object back to squeeze precariously against the other hand or body. No thumb involvement. D. Palmar grasp: Fingers on top surface of object press it into center of palm with thumb adducted.

C

An OT working with a 6 month old can expect grasping of a pellet skills according to Erhardt Prehension Developmental Levels to be: A. No attempt to grasp, but visually attends to object B. inferior-scissors grasp: raking object into palm with adducted totally flexed thumb and all felxed fingers, or two parially extended fingers C. Raking and contacting object D. Pincer grasp: between distal pads of thumb and index finger, distal thumb joint slightly flexed, thumb opposed.

C

An OT working with a 6-7 month old can expect grasping of the cube skills according to Erhardt Prehension Developmental Levels to be: A. Radial-digital grasp: Object held with the opposed thumb and fingertips, space visible with wrist extended. B. Palmar grasp: Fingers on top surface of object press it into center of palm with thumb adducted. C. Radial Palmar grasp: Fingers on far side of object press it against opposed thumb and radial side of palm. with wrist straight. D. Progression toward palmar grasp. Infant's thumb begins to adduct with fingers pressed against ulnar side of palm.

A

An OT working with a 7 month old can expect grasping of a pellet skills according to Erhardt Prehension Developmental Levels to be: A. Inferior-scissors grasp: raking object into palm with adducted totally flexed thumb and all felxed fingers, or two parially extended fingers B. Scissors grasp: Between thumb and side of curled index finger, distal thumb joint slightly flexed, proximal thumb joint extended. C. Pincer grasp: between distal pads of thumb and index finger, distal thumb joint slightly flexed, thumb opposed. D. No attempt to grasp, but visually attends to object

B

An OT working with a 8-9 month old can expect grasping of the cube skills according to Erhardt Prehension Developmental Levels to be: A. Palmar grasp: Fingers on top surface of object press it into center of palm with thumb adducted. B. Radial-digital grasp: Object held with the opposed thumb and fingertips, space visible with wrist extended. C. Radial Palmar grasp: Fingers on far side of object press it against opposed thumb and radial side of palm. with wrist straight. D. Progression toward palmar grasp. Infant's thumb begins to adduct with fingers pressed against ulnar side of palm.

C

An OT working with a 9 month old can expect grasping of a pellet skills according to Erhardt Prehension Developmental Levels to be: A. Fine Pincer grasp between fingertips or fingernails, distal thumb joint flexed. B. Raking and contacting object C. Inferior pincer grasp: between ventral surfaces of thumb joint extended, beginning of thumb opposition D. Pincer grasp: between distal pads of thumb and index finger, distal thumb joint slightly flexed, thumb opposed.

D

An OT working with an 8 month old can expect grasping of a pellet skills according to Erhardt Prehension Developmental Levels to be: A. Fine Pincer graspL between fingertips or fingernails, distal thumb joint flexed. B. Inferior pincer grasp: between ventral surfaces of thumb joint extended, beginning of thumb opposition. C. Inferior-scissors grasp: raking object into palm with adducted totally flexed thumb and all felxed fingers, or two parially extended fingers D. Scissors grasp: Between thumb and side of curled index finger, distal thumb joint slightly flexed, proximal thumb joint extended.

D

An OT working with an individual in a halfway house is determining an individual's ability to complete specific work samples. Which assessment uses work samples that progress from simple to complex, are appropriate to the worker's area(s) of interest, and can be compared to norms which were obtained for persons with disabilities. Which assessment was used? A. Vocational Interest Inventory - Revised B. Jacob's Prevocational Assessment (JPVA) C. Vocational Interest, Temperment, and Aptitude System (VITAS) D. Testing Orientation, and Work Evaluation in Rehabilitation (TOWER)

C

An OTR is advising a client who has had a flexor tendon repair on the timing for resuming ADLs. During what time period is the flexor tendon repair the weakest and most likely to rupture? A. 1-3 days postsurgery B. 4-9 days postsurgery C. 10-12 days postsurgery D. 4-8 weeks postsurgery

C (Injury to the posterior spinal cord generally does not produce motor deficits of the severity produced in other areas of the spinal cord. This is because the posterior spinal cord carries primarily sensory information to the brain. Therefore, motor function is preserved below the area of involvement, even in the presence of profound sensory deficits.)

An OTR is doing an initial evaluation on a patient who has been diagnosed with posterior cord syndrome. Which of the following would the OTR expect to observe with this patient? A. Complete loss of muscle strength below the level of injury, pain and temperature loss, proprioception and touch preserved. B. Muscle weakness below the level of injury, pain and temperature loss, proprioception and touch preserved. C. Muscle strength is preserved, loss of proprioception + vibration sensation + loss of two point discrimination +loss of light touch D. Muscle strength is preserved, pain and temperature loss, proprioception and touch preserved.

A (Pillar pain is pain on either side of the carpal tunnel release surgery site. The source of the pain is unknown and may be ligamentous or muscular in origin.)

An OTR is working with a client who has undergone a carpal tunnel release. The client describes postoperative pain on either side of the carpal tunnel that makes it difficult to grasp objects. What is the term for this type of pain? A. Pillar pain B. Chronic pain C. Phantom pain D. Complex regional pain

A (Symptoms of anterior spinal cord syndrome are normally complete loss of muscle strength below the level of injury. In most patients sensitivity to pain and temperature are lost whilst sensitivity to touch and proprioception (joint position) are preserved.) (Knobs located on the front of the stove would be important, but cooking is an IADL and bathing is an ADL.)

An OTR working with a client with anterior cord syndrome is doing a home evaluation before the client is discharged home. Which of the following is MOST important for the OT to assess? A. Water temperature B. The apartment's capacity to support an emergency call button C. Wheelchair accessibility D. Knobs located on the front of the stove.

B (When the seat back height is above the lower angle of the scapula, it can prevent adequate shoulder extension, which is necessary for contacting the pushrim.)

An OTR® and a Level II Fieldwork student at a long-term care facility observe a new resident moving his wheelchair forward by gripping the front of the pushrim and giving it multiple tiny pushes. The OTR® asks the student, "Why do you think he is pushing his wheelchair that way?" The student observes a bit longer and responds: A. The seat back is too low. B. The wheelchair is too wide. C. The seat back is too high. D. The armrests are too low.

C (In the plateau phase, symptoms are at their most severe: Edema and weakness are common symptoms with GBS. Because symptoms are typically symmetrical and ascend from distal to proximal, bilateral symptoms would likely affect lower-body self-care.)

An OTR® evaluates a client who has just entered the plateau phase of Guillain-Barré syndrome (GBS). When assessing ADLs, what would the OTR® most likely expect to see? A. Bowel and bladder incontinence, which affects skin integrity and toileting routine B. Unilateral sensory impairments, which affect fine motor tasks such as oral care C. Edema and weakness in the lower extremities, which makes bathing and dressing difficult D. Shoulder weakness, which results in decreased independence in upper-body bathing and dressing

D

An OTR® evaluating a client listens as the client says, "My pain is really bad, forcing me to stay in bed 24/7, and I am not able to take care of myself at all." Which clarifying response is BEST? A. "It appears that your pain is really bad, forcing you to stay in bed." B. "I feel badly that you are experiencing pain 24/7, forcing you to stay in bed." C. "You poor thing. That's awful! Where is the pain?" D. "It seems that your pain is so bad you can't get up to go to the bathroom or get yourself food from the kitchen."

A

An OTR® has been treating a client with a distal radius fracture. Because normal pain-free range of motion (ROM) may not be possible after this injury, the OTR has refocused efforts on gaining pain-free motion within the client's functional ROM. How would the OTR determine this client's functional ROM? A. Assist the client in identifying activities the client wants or needs to be able to accomplish with the affected extremity and incorporate practice of these specific activities B. Measure passive ROM to determine the available range of the affected extremity and then focus on place and hold exercises within that available range C. Review the evidence to determine specific measurements that correspond to functional ROM and then formulate functional ROM goals for the client D. Use goniometric measurements of the unaffected extremity to create ROM goals for the affected extremity

Motor

Cranial nerves III, IV, VI are pure _______________: they control eye movements and pupillary constriction. For vision.

B (Sensory integration techniques emphasize the use of naturalistic environments for intervention. Using a small classroom where two other children are doing handwriting tasks is the most naturalistic environment similar to an actual classroom without unnecessary sensory stimulation.)

An OTR® has been working on increasing the concentration and attention span of an 8-year-old child with autism who is distractible during handwriting activities in the classroom. Using a sensory integration approach, which environment is the BEST to address concentration and attention skills? A. An isolated room free from any kind of distraction B. A small classroom with two other children and soothing music C. A small corner of a sensory gym where two other children are swinging D. A quiet corner in the child's classroom with a chalkboard divider

A (According to the FIM score, the patient requires moderate assistance for performing activities. In the acute phase of Guillain-Barré syndrome, the patient may be experiencing muscle belly tenderness and overall fatigue; energy conservation techniques will allow the patient to engage more productively in daily self-care tasks.) (B: In the acute phase of Guillain-Barré syndrome and with a FIM score of 3, the patient will likely be unable to reach three sets of progressive resistive upper-extremity exercise. C: Progressing from a FIM score of 3 to 7 is unrealistic in the acute phase of Guillain-Barré syndrome. D: Guillain-Barré syndrome typically results in more extreme distal muscle weakness rather than proximal muscle weakness, so addressing trunk control will not be an effective intervention.)

An OTR® has completed an evaluation of an inpatient who has acute Guillain-Barré syndrome. The patient scored a "3" on all subtests of the Functional Independence Measure. Based on this information, which outcome would be realistic for this patient to achieve prior to transitioning to a skilled nursing facility? A. Uses energy conservation techniques during daily self-care tasks B. Completes three sets of a progressive resistive upper-extremity exercise program C. Independently completes wheelchair transfers using proper body mechanics D. Demonstrates controlled dynamic trunk movements during IADL activities

C

An OTR® has completed the Allen Diagnostic Module with a client who has dementia. Results indicate that the client is functioning at a 4.6 level (Goal-Directed Actions). Based on this information, which strategy would MOST effectively promote the client's success during a grooming task? A. Demonstrate and ask the client to imitate the OTR®'s movements. B. Guide the client through the physical movements of the task. C. Lay out the items needed for the task prior to the start of the session. D. Use verbal praise during each task to encourage on-task behavior.

C (Phenomenological study design would allow for the collection of participant's experiences through focus groups that contain open-ended interview questions.)

An OTR® has had many observations of the adjustment that clients go through following spinal cord injury. The OTR® would like to put together focus groups to explore the experiences of individuals who are adjusting to spinal cord injury. What study design methodology would be best for this type of research project? A. Randomized controlled trial B. Single-case design C. Phenomenology D. Participant observation

C

An OTR® in a school system is evaluating a 7-year-old child with behavioral and depressive symptoms. Which tool can the OTR® BEST use to establish an occupational profile for this client? A. Social Skills Rating Scale B. Occupational Therapy Psychosocial Assessment of Learning C. Canadian Occupational Performance Measure D. Test of Playfulness

B (Motivation is a big factor for change to occur. Creating a sense of teamwork allows for mutual support to accomplish goals together.)

An OTR® is adapting the environment to reduce the incidence of repetitive strain injuries among office workers. To maximize the effectiveness of the ergonomic program, what would the OTR® be MOST likely to include? A. Visual reinforcement with signs posted around the office and workstations B. Empowerment of the workers and creation of a positive group-safety culture C. A series of lectures and videos showing bad versus good workstation ergonomics D. Education of supervisors in how to identify noncompliant workers

C (To avoid either conflict of interest or the perception of conflict of interest, disclosure to clients and employers of an outside business relationship is advised by the AOTA Ethics Commission.)

An OTR® is also a part-time salesperson for an adaptive equipment vendor. The OTR accepts a position in a rehabilitation hospital as a direct service practitioner and intends to keep the sales job, too. What should the practitioner do, consistent with the AOTA Ethics Commission's advice about engaging in business transactions with clients and ensuring ethical responsibility in a sales position? A. Avoid recommending the purchase of adaptive equipment sold by the vendor the OTR works for B. Sell the equipment to clients but reduce the cost by the amount of the OTR's sales commission C. Disclose the relationship with the outside business to clients and the employer D. Quit the job as an adaptive equipment salesperson

A

An OTR® is asked to consult with a local meat packing plant to establish a work injury prevention program because of an increase in the number of work-related musculoskeletal disorders among employees. For the program to be successful, what should the OTR® FIRST ensure? A. Complete employee participation B. Projection of new injuries C. Anticipated reoccurring injuries D. Long-term beneficial outcomes

C (Confrontation, in which the practitioner identifies the client's behavior in an effort to help the client self-manage it, is an appropriate interpersonal strategy when dealing with clients with mental illness. Confrontation may include limit setting.)

An OTR® is assessing a client who is in an active state of mania. The client has difficulty concentrating on the assessment procedure and displays disruptive behavior, telling the OTR that the OTR is "stupid" and "asking dumb questions." The OTR tells the client, "I would like to help you, but I will not tolerate this behavior. You are not being nice to me right now, and I deserve to be treated better than this." Which interpersonal strategy does the OTR's statement exemplify? A. Validation B. Limit setting C. Confrontation D. Reframing

B

An OTR® is assessing a new client who has a distal radius fracture. Functional outcome reporting is required by the facility. Which approach to functional outcome measurement is MOST consistent with client-centered care? A. Quantitative range of motion measurements, to ensure that quantitative measures are used to facilitate functional return B. A range of quantitative and qualitative measures, to assess clinical outcomes C. Standardized assessment tools, to ensure payment by third-party payers D. Qualitative assessments such as the Disabilities of the Arm, Shoulder and Hand (DASH), as a disease-specific measure for the upper extremity

B

An OTR® is attending an individualized education program (IEP) team planning for a 10-year-old child with autism. Which principle BEST contributes to a good IEP? A. Set academic, functional, and measurable goals achievable within a 6-month time frame B. To the maximum extent possible, the student must be educated with nondisabled peers C. Transition planning must begin with an emphasis on future vocational goals and independent life skills D. To be succinct, the IEP reports only the needs and challenges of the student

B (Resting the arms on the thighs releases the diaphragm, making breathing easier and lessening fear. Pursed lip breathing can slow breathing and reduce anxiety.)

An OTR® is conducting a treatment session with a client with chronic obstructive pulmonary disease. The client is engaged in a light work task while sitting, suddenly experiences extreme shortness of breath, and panics. Which response by the OTR® would be MOST appropriate? A. Ask the client to lean backward and rest the arms on the chair arms, then practice pursed lip breathing B. Ask the client to lean forward and place the forearms on the thighs, then practice pursed lip breathing C. Ask the client to sit upright and allow the arms to dangle and guide the client to use visual imagery D. Ask the client to sit upright, use one arm for the work task, and use the other arm to bear weight and support the trunk

A

An OTR® is developing a job-finding program to assist clients who have an enduring mental illness. How will research findings about supported employment assist the OTR® in the program development process? A. Enables selection of intervention methods based on objective investigation B. Promotes the core values of occupational therapy and its roots in the behavioral sciences C. Supports practice techniques perceived to be beneficial for clients with a mental illness D. Provides evidence to resolve practice issues with administrators and reimbursement sources

A (When a client is on the cardiac care unit, activities should not exceed 1 to 2 METs. Any higher MET level requires too much exertion, which results in increased oxygen consumption and a higher heart rate than may be tolerated.)

An OTR® is developing an intervention plan for a client with postacute myocardial infarction on the cardiac care unit. What metabolic equivalent of task (MET) should the client's early activity intervention not exceed? A. 1-2 METs B. 3-4 METs C. 5-6 METs D. 7-8 METs

B

An OTR® is evaluating a 16-year-old adolescent with major depression and suicide precautions in an inpatient facility. Which source of information will BEST enable the OTR® to document the intensity, severity, and duration of this client's symptoms? A. Observation during occupational performance B. A life events inventory of the client C. Interviews with caregivers D. Interviews with roommates in the facility

A (For a client with an electrical burn, it is especially important to complete gross sensory screening of the involved limb to identify the extent of peripheral sensory nerve involvement. Similarly, for full-thickness and subdermal burns, it is likely that the peripheral sensory nerves are involved. Edema may not be measureable due to bulky dressings)

An OTR® is evaluating an electrician who sustained an electrical burn in the right palmar area 2 days ago. The client has a subdermal burn in the middle of the palm with a full-thickness burn extended into the middle and index fingers. After completing the initial goniometric measurements of the right wrist and fingers, what other assessment is MOST IMPORTANT for the OTR® to perform before developing an intervention plan? A. Gross sensory screening B. Edema measurement C. Grip and pinch strength D. Manual muscle testing

A (Transitional programs include monitoring the client's progress and reassessing performance to determine when job tasks can be upgraded to full time or full duty.)

An OTR® is explaining to a client who was recently evaluated the value and benefit of a transitional work program. Which of the following components would be included in a transitional work program? A. Client's performance of work conditioning activities in the clinic followed by performance of worksite light-duty tasks. B. Client's exploration of options for full-time employment while participating in work conditioning program. C. Client's return to full-time employment while performing at modified work capacity. D. Client's return to part-time employment, performing all required physical job demands.

B

An OTR® is in the process of completing a functional capacity evaluation (FCE) on a client employed as a cashier in a lumber yard. The client was referred to occupational therapy after sustaining a ruptured biceps while lifting a sheet of lumber. The OTR® suspects the client is exhibiting less than maximum effort during assessment. How would the OTR® BEST assess grip strength to determine whether client is exhibiting maximal effort? A. Have the client complete three trials on the second handle setting of the Jamar dynamometer. B. Have the client complete one trial on each of the five handle settings of the Jamar dynamometer. C. Have the client perform rapid exchange between the left and right hands for a total of 20 trials for each hand using the Jamar dynamometer. D. Have the client perform two trials on each of the five handle settings of the Jamar dynamometer and compare the results with normative data.

D (Clients with emphysema have hyperinflated lungs, which partially expand the ribcage.)

An OTR® is interviewing a new client who physically presents with a "barrel chest." This appearance is most often associated with what condition? A. Cystic fibrosis B. Asthma C. Collapsed lung D. Emphysema

A (A closed route is a contained environment that allows a client to perform tasks and respond to car handling or maneuvers.)

An OTR® is observing a client's driving performance in a contained environment; the client is performing tasks and responding to demands involving car controls and car handling or maneuvers. Which testing approach is the OTR® using? A. Closed route B. Simulator testing C. Instrumented vehicle D. Naturalistic driving

B (Antitippers reduce the likelihood of tipping the chair backward during weight shifting, and because the client no longer has support through the lower extremities by resting the foot on the floor, balance when repositioning must be considered.)

An OTR® is ordering a wheelchair for a client with bilateral transfemoral amputations. The client does not have prostheses and has no intention of using prosthetic limbs in the future. Based on this information, which design feature would be MOST BENEFICIAL to include as part of the wheelchair prescription? A. Backrest with vertical zipper B. Anti-tipping attachments C. 5-inch (8-cm) diameter pneumatic front casters D. Adjustable cushioned arm rests

D. (The extensor dorsal hood lies superficially at the proximal interphalangeal joint and will likely be damaged in a full-thickness burn to the dorsum hand. Until the integrity of the extensor dorsal hood is verified, Boutonniére precaution should be applied during evaluation and intervention.)

An OTR® is performing an initial evaluation of a client with an acute full-thickness burn to the dorsum hand involving all digits. The OTR® is applying Boutonniére precaution and avoiding having the patient form active or passive composite flexion of the fingers. What is the MOST APPROPRIATE clinical reasoning for this approach? A. The tensile strength of the burned skin will not allow for composite flexion of the fingers. B. A bulky dressing on the hand will limit the ability to form composite flexion of the fingers. C. Immobilization is critical for the initial wound healing to avoid wound widening from movement. D. Composite flexion of the fingers increases the risk of extensor dorsal hood disruption.

A (A correctly addresses hand weakness and its effects on fine motor tasks of dressing. The biomechanical frame of reference is a common theory used in practice and focuses on restoring or remediating skills to enhance occupational performance. A hand-strengthening program would restore needed ROM and muscle power to adequately perform fine motor tasks.)

An OTR® is planning intervention approaches for a client in the recovery stage of Guillain-Barré syndrome (GBS). According to the biomechanical frame of reference, which preparatory method is the BEST for addressing upper-body dressing? A. Instruct the client in hand-strengthening exercises using therapy putty to increase strength for manipulating fasteners on shirts. B. Measure the client for resting hand splints to wear at night to prevent contractures while ROM returns to bilateral hands. C. Teach the client how to use a dressing stick and button hook to don a formal shirt independently. D. Develop a home exercise program for the client for upper-body ROM and light strengthening and provide education on energy conservation and the importance of rest breaks.

D (Presenting intervention outcomes is important in establishing program justification such as expanding occupational therapy services to include patients who have respiratory disorders.)

An OTR® is presenting an inservice to case managers about the benefits of occupational therapy for patients who have respiratory disorders. What should be the PRIMARY focus of this presentation? A. Differences between occupational therapy and respiratory therapy B. Specific occupational therapy protocols for patients who have respiratory disorders C. Revenue-generating potential of occupational therapy services D. Impact of occupational therapy services on patients' abilities to function at home

A (The Ecology of Human Performance model aims to focus on the interaction of the person, task, and context; therefore, interventions that modify any of these three falls within this model.)

An OTR® is providing home health intervention for a client who recently had bilateral transradial amputations. The client is now learning to use newly acquired prostheses that have voluntary-opening terminal devices. The client's goal is to resume former home maintenance roles. Using the Ecology of Human Performance model, what should be the PRIMARY focus of the intervention? A. Modification of the environment, tools, and tasks for completion of daily activities B. Engagement in graded desensitization activities for the residual limbs C. Instruction about anxiety management techniques to control posttraumatic stress D. Provision of strengthening and endurance activities specific to activity demands

C

An OTR® is providing intervention in the home of a client who is recovering from Guillain-Barré syndrome. The client ambulates independently using a walker, but becomes unsteady when tired. What should the OTR® include as part of the intervention for supporting the client's safety at home? A. Talk with family members about providing contact-guard assistance whenever the client is engaged in daily tasks. B. Advise the client to complete self-care tasks while seated in a wheelchair in the home bathroom. C. Problem-solve with the client to identify and eliminate existing fall risks within the home. D. Teach the client diaphragmatic breathing techniques to use during functional ambulation at home.

A

An OTR® is providing intervention to a medically stable client who sustained upper-extremity partial-thickness burns of the dominant arm, 5% of the total body surface area, 2 days ago. Which intervention BEST represents a typical ADL intervention? A. Instruct the client in the use of a long-handled spoon and fork and a built-up-handled knife for self-feeding. B. Instruct the caregiver to assist the client in self-feeding and grooming tasks to prevent pain with movement. C. Instruct the client in donning and doffing a pressure garment sleeve after applying lotion to the arm. D. Encourage the client to independently self-feed without the use of adaptive equipment.

B (According to Dunn's model, low registration of stimuli is demonstrated by sensory seeking.)

An OTR® is reading a previous initial evaluation report for a child with autism that concluded, on the basis of a sensory processing assessment, that the child shows signs of low registration of sensory information. Which of the child's behavior observed by the OTR® is consistent with low sensory registration? A. Easily gets dizzy with linear swinging B. Shows increased hyperactivity and movement C. Dislikes being touched and hugged D. Shows guarded and cautious movement

C

An OTR® is reviewing the medical chart of a client who has Parkinson's disease. The neurologist has indicated that the client has a festinating gait. What does the client's gait look like? A. The client's steps are stiff and slow. B. The client's steps are halting and unsteady. C. The client's steps are small and rapid. D. The client's steps are marked by pauses.

D

An OTR® is running a group in an innovative preventive program for parolees. The goal of the program is to assist parolees with establishing or restoring occupational balance and productive occupations. The current group has been meeting regularly for 6 months. The members have good insight, and they understand the goals of the program. What leadership style should the OTR use in this setting? A. Directive B. Facilitative C. Cooperative D. Advisory

D (A disorganized, random scanning pattern is characteristic of unilateral neglect, which can be further assessed using a cancellation, drawing, or reading task.)

An OTR® is screening an older adult who has had a stroke. The physician's referral notes that the client eats food on only half of the plate and completes grooming tasks swiftly, making many mistakes. Which scanning pattern would be MOST likely to prompt the OTR to recommend additional testing for unilateral neglect? A. Organized, symmetrical scanning B. Rescanning with an organized scanning C. Abbreviated or shortened scanning D. Disorganized, random scanning

C (Accommodation is the ability of the eye to adjust focus at different distances. Accommodative facility is the speed of focus adjustment and ability to maintain focus over time. Driving a car would be most difficult for this client because looking at the speedometer and then at the road requires adjusting focus at different distances (near distance for the speedometer and farther distance for the road))

An OTR® is selecting intervention activities for a young adult client with a diagnosis of accommodative infacility. Which activity will be MOST difficult for this client? A. Watching sports B. Reading a book C. Driving a car D. Watching a play

A (To elevate onto the large rear tire, it is necessary to grasp the hand rim as far posteriorly as possible and quickly pull forward.)

An OTR® is teaching a client how to pop a wheelie in a manual chair so that the client can negotiate curbs. What instructions would the OTR give this client? A. Grasp the hand rims posteriorly and pull them forward forcefully and abruptly, throwing the head back forcefully, if needed. B. Grasp the hand rims anteriorly and pull them forward forcefully and abruptly, throwing the head forward forcefully, if needed. C. Grasp the hand rims in the center and pull them forward forcefully and abruptly, throwing the head back forcefully, if needed. D. Grasp the hand rims in the center and pull them backward forcefully and abruptly, throwing the head forward forcefully, if needed.

B (With lower-extremity burns, providing vascular support before standing decreases blood pooling in the lower extremity and therefore decreases pain in standing and ambulation. The client should learn an alternative strategy to manage pain early on in the intervention.)

An OTR® is treating a client who burned the bilateral lower extremities 10 days ago. The client refuses to participate in ADLs because of pain. The client rates the level of pain as 4 of 10 when resting in bed and 7 of 10 when standing. What is the BEST action for the OTR® to take? A. Report the client's pain levels to the medical doctor and request stronger pain medication. B. Apply an elastic bandage wrap for vascular support before getting the client out of bed. C. Allow the client to perform ADLs while lying in bed, and focus on bed mobility skill. D. Explain to the client that nonparticipation may lead to an early discharge from therapy.

A (A task-oriented group presents opportunities for participants to practice situations they may encounter in daily life. A group environment in which conflicts are likely to occur can help participants practice resolving conflict in appropriate ways.)

An OTR® is using Fidler's task-oriented group as a context for treatment to help participants explore healthy ways to deal with conflict. Which of the following activities would be BEST for this group? A. An activity that is challenging and likely to cause tension among members B. An activity that is highly structured so participants know exactly what is expected of them C. A parallel group activity that enables participants to work alongside one another D. An activity that facilitates participants' success and thus improves their self-esteem

C

An OTR® is using cognitive-behavioral therapy strategies to help an adolescent client with attention deficit hyperactivity disorder regulate behavior so the client is able to get to work on time. Which strategy is related to cognitive-behavioral therapy? A. Encouraging the client to take a movement break when the client begins to feel off task while getting ready for work. B. Encouraging the client to set an alarm clock so that the client is able to get up for work on time. C. Encouraging the client to visualize completing the prework routine without getting distracted and arriving to work on time. D. Encouraging the client to call the boss when the client is running late instead of sneaking in through the back door.

D

An OTR® is using the Ayres Sensory Integration Intervention program for a 6-year-old child with attention deficit hyperactivity disorder and sensory-seeking behaviors. Which principle should the OTR® keep in mind when selecting activities for this intervention program? A. The sensory environment is completely structured to allow for optimum integration of sensory information. B. Sensory experiences are limited to one sensory factor at a time to ensure mastery. C. Passive participation on the part of the child is encouraged. D. Appropriate activities are done individually and not in a small group.

C (Because a donor heart is denervated, the autonomic nervous system does not control the client's heart rate. The heart relies on circulating hormones, which take longer to increase and decrease the heart rate.)

An OTR® is working in a cardiac rehabilitation program. Of the four clients on the OTR's caseload, which client would require a longer warm-up and cool-down period during exercise and activity sessions? A. The client with congenital heart disease B. The client with a coronary artery bypass graft C. The client with a heart transplant D. The client with an automatic defibrillator

B (A multidisciplinary team is an interprofessional team composed of individuals representing the professional disciplines that serve the client. A: Transdisciplinary teams function without discipline-centered boundaries. C: Interdisciplinary teams set goals and engage in intervention collaboratively across the disciplines. D: Allied health teams are not a recognized, definable type of team collaboration.)

An OTR® is working in an inpatient setting with veterans who have upper-extremity amputations. Each morning the rehabilitation team, consisting of physician, nurse, psychologist, social worker, OTR, physical therapist, and orthotist, come together to discuss their patients' clients' progress toward the goals set within their individual disciplines. What type of team is this? A. Transdisciplinary team B. Multidisciplinary team C. Interdisciplinary team D. Allied health team

A

An OTR® is working in the school system with a child who has a behavioral disability. The OTR® wants to improve the child's classroom environment to promote more opportunities for success. Which rational intervention (RI) approach would the OTR® use? A. Facilitation B. Monitoring C. Gentle correction D. Reprimand

A (Children with congenital heart defects may want to be independent in play and self-care tasks. Pacing activities and selecting appropriate activities are two ways to promote their independence.)

An OTR® is working on an inpatient unit with a child who has a congenital heart defect. Before admission, the child was able to complete a basic self-care routine. Currently, despite wanting to complete the activities independently, the child requires assistance because of compromised endurance. What compensatory strategy or technique should the OTR® use to minimize the impact of the condition on the child's occupational performance? A. Pacing of activities B. Backward chaining of activities C. Forward chaining of activities D. Delegating activities

D

An OTR® is working on cooking skills with a client with a severe intellectual disability. Which activity would the client likely be able to complete with supervision? A. Making a frozen pizza in the oven. B. Preparing a cold salad. C. Heating up a microwave meal. D. Pouring juice from a pitcher to a cup

A (People with achondroplasia (often referred to as dwarfism) usually grow be to 4 feet tall or less in height.)

An OTR® is working on life skills with a teenager who has achondroplasia. Which of the following tasks might pose a challenge for the client? A. Reaching an upper cupboard to obtain a box of cereal B. Transporting silverware from the dishwasher to the silverware drawer C. Sweeping the kitchen floor after a meal D. Making toast using a toaster

C (An arterial line catheter is inserted in the radial artery at the wrist to continuously monitor arterial pressure. When the wrist is moved, it can disrupt the catheter and affect the blood pressure reading.)

An OTR® is working on the intensive care unit of a hospital. The client has hypertension and has an arterial line catheter inserted in the radial artery. What change in the client is the OTR MOST likely to record during treatment? A. A decrease in blood pressure when the head of the bed is elevated B. A decrease in heart rate when client is positioned from supine to short sitting at the edge of the bed C. An inaccurate blood pressure reading when the wrist is moved D. An increase in respiration rate when the client is positioned side lying

B

An OTR® is working on the organ transplant unit of a major hospital. What psychiatric condition is a client with a long-term transplantation MOST likely to experience? A. Depression B. Anxiety C. Dementia D. Personality disorder

A

An OTR® is working with a 3-year-old child with autism and oral defensiveness to improve feeding skills. Which activity is BEST to begin desensitization of this child's oral defensiveness? A. Make the child hold a vibrating toy and put it close to the face B. Provide deep pressure on distal parts of the body and slow linear rocking C. Ask the child to touch the lips with a piece of cracker D. Provide only preferred foods during feeding sessions

A (An extreme startle may be associated with a retained Moro reflex.)

An OTR® is working with a child who has a diagnosis of cerebral palsy. The child demonstrates an extreme startle response on hearing a loud noise coming from another treatment space. Which key marker of cerebral palsy is this behavior associated with? A.Retention of primitive reflexes B. Abnormal or variable tone C. Clonus D. Involuntary movements

B

An OTR® is working with a client who fractured the distal radius 6 weeks ago. Within what time frame can controlled AROM be initiated if the fracture is healing secondarily? A. 0-3 weeks postinjury B. 3-6 weeks postinjury C. 6-9 weeks postinjury D. 9-12 weeks postinjury

D (Ventricular tachycardia of more than 100 beats per minute can cause sudden cardiac death. Therapy should be deferred until the client is medically stable.)

An OTR® is working with a client who has ventricular tachycardia. The client's vital signs include a heart rate greater than 100 beats per minute. What is the appropriate therapeutic response to this client's situation? A. Allow the client to rest 10 minutes, then begin an occupational task that focuses on upper-extremity movement. B. Work with the client in a supported sitting position with the client's legs elevated. C. Instruct the client in adapting performance of grooming and feeding tasks. D. Defer the client's participation in occupational therapy until later, because the client is medically unstable.

B

An OTR® is working with a client who is planning to take fixed-route transportation from home to school. The client sustained a closed-head injury 2 years ago and has difficulty with problem solving, especially related to changes in routine. What is the MOST important step for the OTR® to practice with the client? A. Walking from home to the train station B. Learning how to read the train schedule C. Obtaining exact change for the fare D. Getting on and off the train safely

A (B: Unstable angina does not resolve with rest or nitroglycerin. It requires emergency treatment.)

An OTR® is working with a client who suddenly complains of pressure and tightness in the chest. The client's symptoms resolve with rest and a nitroglycerin tablet administered under the tongue. What condition is the client MOST likely to have experienced during the treatment session? A. Stable angina B. Unstable angina C. Myocardial infarction D. Atrial flutter

D (Clients with neglect or visual attention deficits are usually anosognostic and do not gain insight into their impairment, or the effect of the impairment, on driving performance.)

An OTR® is working with a driver who is experiencing visual neglect poststroke. What is the MOST APPROPRIATE compensatory approach the OTR® can use to help the driver improve on-road performance? A. Teach the person to look at a reference point in the vehicle, such as the center of the hood, to improve lane maintenance. B. Avoid night driving to limit exposure to the bright lights from other vehicles C. Avoid driving in peak traffic hours and in inclement weather. D. Awareness training pending the client's insight.

A (Visual discrimination is the brain's ability to process and interpret the features of an object (or other stimuli) related to matching, recognition, and categorization by different attributes.)

An OTR® is working with a pediatric client with delayed visual-perceptual skills. Specifically, the child has difficulty with shape, letter, and color identification and recognition. On the basis of this description, with which visual-perceptual skill is the child having difficulty? A. Visual discrimination B. Visual memory C. Visual closure D. Figure-ground recognition

D (The experimental study design addresses intervention effectiveness because participants are assigned to different groups to determine the effects of the intervention.) (A, B: Case-control and correlational designs are used to identify relationships between variables to identify risk factors or causes of a situation. The use of two groups with different interventions requires effectiveness measures over time. C: Descriptive study design is used to describe and analyze a phenomenon, not determine the effectiveness of an intervention.)

An OTR® reads a scientific study that compares treatment effectiveness after an activity-based intervention versus rote exercise to increase upper extremity function in two groups. The mean score on the activity-based group increased by 14 points, and the mean score of the rote control group increased by 12 points. Results from the t test that compared the group outcomes were statistically significant (p < .05). Which type of evidence BEST describes the results of this scientific study? A. Case-control B. Correlational C. Descriptive D. Experimental

C (Attention deficit hyperactivity disorder (ADHD) often occurs with oppositional defiant disorder (ODD), so practitioners conducting an assessment with a client with ODD should always look for signs of ADHD.)

An OTR® receives a referral for a 7th grader diagnosed with oppositional defiant disorder. Which condition that often occurs concurrently with oppositional defiant disorder should the OTR® look for? A. Depression B. Conduct disorder C. Attention deficit hyperactivity disorder D. Schizophrenia

D (A comprehensive FCE should assess all the physical demands of work as defined by the Dictionary of Occupational Titles (DOT). The DOT serves as a source document for job demands and aptitudes for all job classifications. OTR®s should refer to DOT and the employer-provided job description to identify essential job functions.)

An OTR® receives a referral to complete a functional capacity evaluation (FCE) on a client 2 months after a Stage 2 quadriceps strain. The client is employed as a police officer and sustained the injury during an altercation with a suspect. To identify the client's essential job functions, which sources would be BEST for the OTR® to use? A. The U.S. Department of Labor's Dictionary of Occupational Titles, O*Net, job classifications, and client interview B. Employer-provided job description, client and supervisor interview, and completion of a vocational evaluation C. The National Institute of Occupational Safety and Health (NIOSH) guidelines, job description, and client interview D. The U.S. Department of Labor's Dictionary of Occupational Titles job classifications and job description

C

An OTR® receives evaluation orders for a client who has recently experienced traumatic brain injury. The client displays severe memory impairment and can only respond to simple commands without being distracted. What Rancho Los Amigos level BEST corresponds with the client's behavior? A. VII B. III C. V D. II

A (The ADReS is the composite battery developed by the American Medical Association for physician administration.)

An OTR® recommends that a physician conduct a 10-minute in-clinic screening to address a variety of driving-related performance components. Which composite battery would be the BEST choice for such a screening? A. Assessment of Driving Related Skill (ADReS) B. Driving Health Inventory C. Occupational Therapy Driver Off-Road Assessment D. Roadwise Review

C (Cross-out assessments are used in visual scanning and can be beneficial in detecting spatial neglect or visual field impairments.)

An OTR® reviews the cancellation test form in a client's chart. The form indicates that the client crossed out all letter Ms. What does this test assess? A. Visual acuity B. Literacy C. Spatial neglect D. Ocular motor control

D

An OTR® supervises a COTA® who works with a group of adults who are continuing their recovery from alcohol and drug addiction by living in a halfway house. The OTR® suggests that the COTA® become familiar with a particular group intervention in working with these clients. What type of intervention would the OTR® MOST likely suggest? A. An occupational deprivation group B. ProjectMAINSTREAM C. A heavy work activity group D. A 12-step self-help group

A (A hallmark of cognitive-behavioral therapy is use of psychoeducation. Evidence supports the benefits of this approach.)

An OTR® uses cognitive-behavioral interventions in psychosocial practice with clients with stress and anxiety disorders. What intervention is the OTR® MOST likely to use? A. Psychoeducation that provides resources and information about the client's illness and ways to cope with it B. Creative expression using art to free underlying emotions and conflicts C. Psychotherapy that builds trust in the client-therapist relationship and brings unconscious conflicts to a conscious level D. Reminiscence to increase awareness of remote memories and thereby improve the ability to recall and recognize

D

An OTR® wants to develop a group activity for clients with personality disorder in an inpatient psychiatric facility. The group has Allen Cognitive Level (ACL) scores ranging from 5.0 to 5.4. What activity would be MOST appropriate to use with these clients in the initial stages of the group? A. Volunteer activities in the community B. Long-term budgeting for house repairs C. Vocational retraining and job seeking D. Role playing social interactions

D (An interactive computer-based program to simulate the activity of driving, also known as a driving simulator, is the best off-road choice because of its relative validity with on-road outcomes.)

An OTR® wants to evaluate a client's fitness to drive. Which off-road assessment would be the BEST for the OTR® to administer? A. A computer-based information-processing training tool B. An off-the-shelf driving-based computer game C. A brake reaction timer used to simulate braking D. An interactive computer-based program to simulate the activity of driving

A (Guided imagery is an evidence-based cognitive strategy that can facilitate improved motor learning.)

An OTR® wants to use a cognitive strategy to address a motor learning deficit with a client. The client is having difficulty with lower body dressing after hip replacement. Which is the BEST example of a cognitive strategy to use with this client? A. The client imagines using a dressing stick while the OTR describes the process of lower body dressing as the client visualizes completing the activity. B. The client completes and repeats one step of the dressing task until the client can do it independently (e.g., putting the affected leg into the pant opening). C. The client uses various garments to practice lower-extremity dressing (e.g., shorts, sweatpants). D. The client practices dressing while on the edge of the bed, on a mat table, or on a chair until the client is comfortable getting dressed from a variety of surfaces.

B

An OTR® working in a public school receives a referral for a middle school student with oppositional defiant disorder. Which method to gather reliable information about this student would be MOST effective? A. Interview the student B. Interview the teacher or caregiver C. Read the student's school file D. Observe the child at home

A

An OTR® working with an adult client with major depression in an inpatient setting assesses the client to determine the impact of occupational and environmental demands on performance of daily occupations. What is most likely to be the OTR®'s PRIMARY theoretical approach to designing an intervention? A. Modifying the environment or the occupation to reduce demands, address personal goals, and use developed skills B. Facilitating age-appropriate occupation through motivation and habit formation C. Exposing underlying conflicts from early childhood relationships that impede current engagement in occupations D. Challenging ineffective adaptive responses and focusing on enhanced occupational adaptation

A (Positioning a client with TBI upright in a wheelchair provides optimal positioning to minimize abnormal tone and increase stimulation of the client's visual and vestibular systems.)

An OTR®; is treating a client who is in a vegetative state after a traumatic brain injury (TBI). The OTR has identified the need to use restorative strategies with the client. Which restorative strategy is MOST appropriate for this client? A. Upright positioning strategies in a wheelchair to normalize muscle tone and facilitate arousal IncorrectB. A sensory stimulation program to facilitate return to consciousness C. A self-feeding program that simplifies the task and provides success for the client D. A behavioral reinforcement program to reinforce the client's on-task performance

D

An adult in a work-hardening program would like to return to work. Which assessment evaluates the psychosocial and environmental factors related to an individual's past work experience, job setting, and ability to return to work? A. Vocational Interest, Temperment, and Aptitude System (VITAS) B. Smith Physical Capacity Evaluation C. Valpar Component Work Sample D. Worker Role Interview (WRI)

A

An adult who is interested in a career change is unsure where he/she would like to begin seeking employment. Which assessment evaluates vocational interests, temperament, and aptitudes to assist with career guidance and vocational placement? A. Vocational Interest, Temperment, and Aptitude System (VITAS) B. Smith Physical Capacity Evaluation C. Valpar Component Work Sample D. Reading-Free Vocational Interest Inventory

A (Collateral and background information from all these sources are necessary to form a comprehensive picture of the client's fitness-to-drive background.)

An entry-level OTR® working in an outpatient rehabilitation clinic is evaluating a client who was recently hospitalized for dehydration and subsequently diagnosed with mild cognitive impairment. The client lives alone in an independent living facility. Before admission, the client was independent in ADLs and light homemaking and active in community and social activities, including driving independently. The client reports no crash record and no violations or citations but avoids night driving, highway driving, and driving in rush-hour traffic or heavy rain. By what means would the OTR® MOST APPROPRIATELY gather background information on the client's fitness to drive? A. Contact the Department of Motor Vehicles to verify the client's driving history, do a chart review, and gather collateral information. B. Do a chart review, and ask for a copy of the client's driver's license. C. Do a chart review, and interview a close relative about the client's driving skills and avoidance behaviors. D. Interview the client about the client's driving experiences and skills.

B (Driving assessment is within the OTR®'s scope of practice, it is an IADL, not an ADL. C: The OTR® should be using clinical reasoning, critical thinking, and best evidence to problem solve whether a client is fit to drive, not just clinical reasoning. D: A comprehensive evaluation, not just assessment of functional performance components, is necessary.)

An entry-level OTR® working in an outpatient rehabilitation clinic is evaluating a client who was recently hospitalized for dehydration and subsequently diagnosed with mild cognitive impairment. The client lives alone in an independent living facility. Before admission, the client was independent in ADLs and light homemaking and active in community and social activities, including driving independently. The client reports no crash record and no violations or citations but avoids night driving, highway driving, and driving in rush-hour traffic or heavy rain. During the multidisciplinary team meeting, the referring physician asks the OTR® to justify why OTR®s are suited to assess this client's fitness to drive. How would the OTR® BEST respond? A. Driving is an ADL included in the scope of practice of occupational therapy and therefore within the OTR®'s domain to assess. B. OTR®s need to extrapolate beyond the walls of the clinic to consider how clients will function in the community; making such an extrapolation, assessing driving fitness is no different from considering whether a client is fit to live alone. C. Occupational therapists use clinical reasoning to make determinations regarding a client's IADLs, including driving. D. OTR®s are trained to use clinical tests to assess domains of function. Because driving entails motor, sensory, cognitive, and visual functioning, occupational therapists may make fitness-to-drive decisions on the basis of functional performance of clients.

B

An individual who avoids or seeks to avoid stimuli presents with a(n) A) passive behavioral response B) active behavioral response

A

An individual who makes no attempt to change the intensity or duration of sensory input presents with a(n) A) passive behavioral response B) active behavioral response

B

An individual with disabilities is interested in clerical work. The OT working with this client assesses the client's basic functional capabilities (e.g upper extremity function, dexterity, visual coordination). The client completed 23 work samples that are administered individually except for the cooperative assembly task. Which assessment did the OT use to assessment was used? A. Worker Role Interview B. Valpar Componenet Work Sample C. Epic Functional Evaluation System D. Vocational Interest, Temperment, and Aptitude System (VITAS)

C

An insurance company evaluates an OT intervention plan that specifies how and why care should be provided in order to approve ongoing therapy. Which of the following reviews is being conducted? A) Concurrent review B) Retrospective review C) Prospective review D) Utilization review

B

An occupational therapist working in a halfway house is assisting a client with identification of areas of interest. The evaluator presents a group of three pictures representing unskilled, semi-skilled, and skilled job tasks, and requests the individual select the picture that represents the job task most preferred. Which assessment is the therapist using? A. Valpar Component Work Sample B. Reading-Free Vocational Interest Inventory C. Vocational Interest Inventory D. McCarron-Dial System

C

An occupational therapist working in early intervention at a preschool observes a child playing for approximately 20 minutes. The therapist notes the child's space management, material management, imitation, and participation. Which of the following assessments is the therapist using? A. Play History B.Test of Playfulness C. Revised Knox Preschool Play Scale D. Test of Playfulness

parasympathetic

Cranial nerves III, VII, IX, X carry p________________ fibers of ANS; involved in control of smooth muscles of inner eye (III), salvatory and lacrimal glands (VII), parotid gland (IX), muscles of the heart, lung, and bowel (X).

A (The suggested interrater reliability of a standardized test is 80%; a point-by-point agreement of 70% is generally too low for good interrater reliability and suggests that there are differences in the way practitioners are administering the assessment.)

An occupational therapy clinic has purchased a new standardized assessment tool. The occupational therapy practitioners are checking the interrater agreement for routinely using the tool. The point-by-point agreement for all raters at the clinic was found to be 70%. What information does this calculation provide? A. Additional practice for administering and scoring the tool is needed. B. Minimum acceptable standards for interrater reliability have been met. C. Service competency for raters using the assessment tool has been attained. D. The majority of raters administered the assessment tool correctly.

B (Macular degeneration results in a gradual loss of ability to see objects clearly. Objects appear distorted in shape and straight lines look wavy or crooked, creating difficulty with reading.)

An older adult client complains that objects appear distorted—for example, straight lines look wavy. The client has difficulty reading and makes frequent mistakes when reading but is able to navigate busy and crowded environments using peripheral vision. Which visual deficit would the OTR® suspect? A. Glaucoma B. Macular degeneration C. Cataract D. Diabetic retinopathy

C

An older adult in a SNF reports he/she is feeling under stimulated and would like more to do. The OT working with the client decides to assess his/her use of leisure time and identify ways leisure can be modified to better fit his/her needs. Which assessment would be the best to administer to this client? A. Minnesota Leisure time Physical Activity Questionnaire B. Leisure Diagnostic Battery (LBD) C. Leisure Satisfaction Questionnaire D. Meaningfulness of Activity Scale

incorrect

Answer choices that are not SMART enough to RUMBA are generally _________________.

A

Are peripheral nerve injuries tested: A) distal to proximal following peripheral nerves B) Proximal to distal following peripheral nerves

D

Aronold-Chiari Syndrome is a form of spina bifida that results from increased pressure due to hydrocephalus (cerebral spinal fluid is not absorbed). This condition is a result of: A) Enlarged ventricles B) Sac rupture C) Portion of the pons slip down through the foraman magnum to the cervical spinal cord D) Portion of the medulla oblongata and cerebellum slip through the foraman magnum to the cervical spinal cord.

C

As trunk stability improves, shoulder flexion with slight external rotation, elbow extension, forearm supination, and slight wrist extension begin to emerge at ______________. A. 4 months B. 2 months C. 9 months D. 6 months

37, 4-6

Asymmetric tonic neck is present at _________ weeks and integrates at _________-________ months

C

At what age can a therapist expect a child to close back zipper, tie bows, button back buttons, snap back snaps? A. 4.5 years B. 5 years C. 6 years D. 3.5 years

B

At what age can a therapist expect a child to cooperate with dressing (holds out arms and feet) pulls off shoes, removes socks, pushes arms through sleeves and legs through pants? A. 2 years B. 1 year C. 2.5 years D. 3 years

D

At what age can a therapist expect a child to find front of clothing, snap or hook front fastener, unzip front zipper on jacket, put on mittens, button a series of three or four buttons, unbuckle shoe or belt, dress with supervision? A. 4 years B. 5 years C. 2.5 years D. 3.5 years

A

At what age can a therapist expect a child to put belt in loops? A. 4.5 years B. 5 years C. 6 years D. 3.5 years

C

At what age can a therapist expect a child to put on a pullover shirt with min A, put on shoes without fasteners ( may be on wrong feet), put on socks (heel may be on top), ind. pull down pants, zip and unzip jacket once on track, needs A to remove pullover shirt, button large front buttons? A. 2.5 years B. 4 years C. 3 years D. 5 years

B

At what age can a therapist expect a child to remove and pull down pants with elastic waist, assist in pulling on socks, put on front-button coat or shirt, and unbutton large buttons? A. 3 years B. 2.5 years C. 1 year D. 3.5 years

B

At what age can a therapist expect a child to remove pullover garment ind., buckle shoe or belt, zip jacket zipper, put on socks correctly, put on shoes with assistance with tying laces, lace shoes, consistently find front/back of garments? A. 5 years B. 4 years C. 3.5 years D. 6 years

C

At what age can a therapist expect a child to tie and untie knots, dress unsupervised? A. 4.5 years B. 6 years C. 5 years D. 4 years

A

At what age can a therapist expect that a child can remove unfastened coat, removes shoes if laces are untied, help pull down pants, find armholes in pullover shirt? A. 2 years B. 1 year C. 2.5 years D. 3 years

16

At what age does transitional planning begin to help a student plan a course of study that will lead to post-school goals? Transition services also begin at this age to provide student with a coordinated set of services to attain post-school goals. Transition plans must be updated annually with appropriate services revision provided.

65

At what age is 20-40% of muscle mass in the nonexercising adult?

20-90

Atrophy of nerve cells in cerebral cortex: over-all loss of cerebral mass/brain wiegh tof 6-11% between ages _____-_____; accelerating loss over age 70.

Mixed

Cranial nerves V, VII, IX, X are _______________: involved in chewing (V), facial expressions (VII), swallowing (IX, X), vocal sounds (X), sensations from head (V, VII, IX), alimentary tract, heart vessels, and lungs (IX, X), and taste (VII, IX, X).

Motor

Cranial nerves XI, XII are pure ________________: innervating sternocleidomastoid, trapezius, and tongue

4-7 years

Creative play occurs between the ages of _____ -_____ ___________. Child engages in sensory, motor, cognitive, and social play experiences in which the child refines relevant skills. Promotes performance in school and work related activities.

C

Creeping: At which age does a child crawl forward on belly? A) 11-12 months B) 10-11 months C) 7-10 months

A

Creeping: At which age does a child creep well? A) 11-12 months B) 10-11 months C) 7-10 months

B

BEERY assesses visual motor integration using geometric forms which are sequences according to level of difficulty. What is the age range for the visual motor and visual perception assessment BEERY? A) birth - 6 months B) 2 - 100 years C) 4-10 years, 11-74 D) 3.5-5.5

A

Back pain injury reports from workers at a bicycle manufacturing company have increased. Management is seeking an OTR®'s recommendations for redesigning the work environment using engineering controls, particularly to reduce awkward postures and repetitions on the assembly line. Which recommendation is the BEST engineering control to redesign the work environment for this organization? A. Place bicycles on a wheeled platform for movement along the assembly line. B. Slow the speed of the assembly line so workers have more time. C. Implement stretch breaks every 45 minutes to improve posture. D. Enforce a work safety policy that requires workers to wear a back support.

7-8 months

Beginning of mastication of soft and mashed foods with diagonal jaw m/m

symbolic play

Between the ages of 2 and 4 _____________ ____________ develops: The child engages in play experiences through which the child formulates, tests, classifies, and refines ideas, feelings, and combined actions.

D

Bilateral integration: as the child experiments with movement, the nervous system is stimulated, and the resulting sensations help the child to coordinate the two sides of the body. This action begins at ________________ A. 5-7 months B. Twelve to twenty-four months C. Eighteen months D. 9-12 months

B

Creeping: At which age does a child perform reciprocal creep? A) 11-12 months B) 10-11 months C) 7-10 months

Cognitive-behavioral

Which frame of reference works on the thoughts and reactions related to environmental triggers. Through journaling and reflection, the client can identify triggers that cause the anxiety to escalate.

B

Which frames of reference would an OT working under the community model be most likely to use? A) Role acquisition and cognitive remediation B) Life-style performance and occupation adaptation C) Biomechanical and neurodevelopmental

A

Crossing the midline: As the child becomes more mobile, movement against gravity and weight shift increase, leading to eventual crossing of the midline, often in an attempt to reach for a toy, while weight bearing on the opposing upper extremity for balance. This action begins at _________________ A. 9-12 months B. 5-7 months C. Twelve to twenty-four months D. Eighteen months

A

Which frames of reference would an OT working under the education model be most likely to use? A) Role acquisition and cognitive remediation B) Life-style performance and occupation adaptation C) Biomechanical and neurodevelopmental

A (children who have mild, moderate, or severe autism)

Childhood Autism Rating Scale assesses the severity of autism. 15 descriptive statements including characteristics, abilities, and behaviors that deviate from the norm. Distinguishes children with autism from children with developmental delays who do not have autism. What is the age range for Childhood Autism Rating Scale (CARS)? A) Over 2 B) 15+ years, 4-36 mos

B (15 + years coping inventory; 4-36 mos early coping inventory)

Coping Inventory and Early Coping Inventory assesses coping habits, skills, and behaviors including effectiveness, style, strengths, and culnerabilities to develop intervention plans for coping skills. What is the age range for Coping Inventory and Early Coping Inventory? A) Over 2 B) 15+ years, 4-36 mos

A

Dunn's neurolgical thresholds and behavioral responses: child who presents with sensory seeking displays... A) Low neurological thresholds and active behavioral responses B) High neurological thresholds and passive behavioral responses C) Low neurological thresholds and active behavioral responses D) Low neurological thresholds and passive behavioral responses

A

Revised Knox Preschool Play Scale includes observation of play to differentiate developmental play abilities, strengths and weakness, and interest areas. Administered in a natural environment with peers. What is the age range for Revised Knox Preschool Play Scale (RKPPS)? A) 0-6 years B) Infancy to 6 years C) Children and Adolescents D) 15 months - 10 years

growth, vomiting

Signs of a blocked shunt in the first year of life include extreme head ________________ and often a soft spot on the forehead. Signs and symptoms by the seoncd year of life include severe headache, __________________, and/or irritability.

B

DeGangi-Berk Test of Sensory Integration (TSI) is a standarized test that measures sensory integrative function with focus on vestibular system: bilteral motor coordination, postural control, and reflex integration. What is the age range for this assessment? A) Elementary aged children B) 3-5 years C) 4-8.11 years D) 1-18 months of age

D

Decreased thought and speech related to schizophrenia is A) Anhedonia B) Anergia C) Affective flattening D) Alogia

A

Denver Developmental Screening Test II assesses a Child's performance in personal-social, fine motor adaptive, language, and gross motor abilities. What is the age range for the developmental assessment Denver Developmental Screening Test II? A) 1 month to 6 years B) 2 yrs 9 mos - 6 yrs 2 mos C) 1 to 42 mos

A

Difficulty in experiencing pleasure is known as A) Anhedonia B) Anergia C) Affective flattening D) Alogia

RUMBA

Documentation that follows realistic, understandable, measurable, behavioral, and achievable are the _________________ form of documentation

SMART

Documentation that follows specific, measurable, attainable, relevant, and time-limited are the __________________ form of documentation

B

Dunn's neurolgical thresholds and behavioral responses: child who presents with poor registration displays... A) Low neurological thresholds and active behavioral responses B) High neurological thresholds and passive behavioral responses C) Low neurological thresholds and active behavioral responses D) Low neurological thresholds and passive behavioral responses

C

Dunn's neurolgical thresholds and behavioral responses: child who presents with senosry sensitivity displays... A) Low neurological thresholds and active behavioral responses B) High neurological thresholds and passive behavioral responses C) Low neurological thresholds and active behavioral responses D) Low neurological thresholds and passive behavioral responses

A

Dunn's neurolgical thresholds and behavioral responses: child who presents with sensory avoiding displays... A) Low neurological thresholds and passive behavioral responses B) High neurological thresholds and passive behavioral responses C) Low neurological thresholds and active behavioral responses D) Low neurological thresholds and active behavioral responses

A (This goal is linked to the child's acquisition of hand skills through engagement in occupational activities and is a reasonable goal in the given time frame.)

During a standardized assessment of developmental motor skills, a 5-year-old child is unable to stack 1-inch (2-cm) cubes as per the standardized instructions. Based on this finding, the OTR® plans to include intervention activities for increasing the child's motor control. Which goal related to this objective would be MOST ACHIEVABLE within a 60-day time period? A. After playing with toys, the child will clean up the play area by placing five small toys in a container with minimal verbal cues. B. The child will consistently fasten the bottom three buttons of a front-opening shirt when prompted during dressing activities. C. The child will score at age level when asked to stack blocks during readministration of the fine motor skills assessment. D. The child will throw a bean bag onto a target with 90% accuracy from 4 feet (1.2 m) away.

A

During an initial evaluation a therapist wants to know a patient's memory function. Which test would be most appropriate? A. Rivermead Behavioral Memory Test B. Montreal Cognitive Assessment (MOCA) C. Assessment of Motor Process Skills (AMPS) D. Allen Cognitive Level Test

C (The Role Checklist gathers information on the client's former and current roles and the value the client places on these roles, consistent with the Model of Human Occupation.)

During an initial interview with a young adult who has Level 1 autism spectrum disorder without intellectual impairment (Asperger syndrome), the OTR® notices that the client lives alone and is isolated, does not go out much, does not value family relationships, and does not seem to have any friends. Additionally, the client is not gainfully employed and does not take care of the apartment. The client spends most of the day surfing the Web and watching TV. Using the Model of Human Occupation, which of the following assessments would be the MOST appropriate? A. A projective test such as House-Tree-Person B. Worker Role Interview C. Role Checklist D. Canadian Occupational Performance Measure

c

During this period of development, child is driven to challenge sensorimotor competencies through roughhouse play, playground activities, games, sports, music, dancing, arts and crafts, household chores, and school tasks. These activities also provide opportunities to promote social development and self-esteem. A. Two to three years B. Eighteen to twenty-four months C. Three to seven years D. Thirteen to twenty-four months

C

During this period of development, refinement as the vestibular, proprioceptive, and visual systems further develop, leading to improved balance and postural control. Further development of tactile discrimination and localization lead to improved fine motor skills. Motor planning and praxis ideation also progress during this period. A. Thirteen to twenty-four months B. Three to seven years C. Two to three years D. Six to twelve months

D

During this period of development, tactile perception becomes more precise allowing for discrimination and localization to further refine fine motor skills. Further integration of all systems promotes complexity of motor planning as the child's repertoire of movement patterns expand. Symbolic gesturing and vocalization promotes ideation, indicating the ability to conceptualize. Motor planning abilities contribute to self concept as the child begins to master the environment. A. Six to twelve months B. Neonatal C. Eight to thirteen months D. Thirteen to twenty-four months

C

During this period of development, tactile, proprioceptive and vestibular inputs are critical from birth onwatd for the eventual development of body scheme. Vestibular system, although fully developed at birth, continues to be refined and impacts the child's arousal level. Helps an infant feel more organized and content. Visual system develops as infant responds to human faces and items of high contrast placed approximately 10 inches from face. Auditory system is immature at birth and develops as the infant orients to voices and other sounds. A. Prenatal period B. first six months C. Neonatal period D. Thirteen to Twenty-four months

B

During this period of development, vestibular, proprioceptive, and visual systems become more integrated and lay the foundation for postural control, which facilitates a steady visual field. Tactile and proprioceptive systems continue to be refined, laying the foundation for development of somatosensory skills. Visual and tactile skills become more integrated as the child reaches out and grasps objects, laying the foundation for eye-hand coordination. Infant movement patterns progress from reflexive to voluntary and goal-directed. A. Neonatal period B. First six months C. Two to three years D. Six to twelve months

A

During this period of development, vestibular, visual, and somatosensory responses increase in quantity and quality as the infant becomes more mobile. Tactile and proprioceptive perceptions become more refined, allowing for development of fine motor and motor planning skills. Tactile and proprioceptive responses also lead to midline skills and eveltual crossing of midline. Auditory, tactile, and proprioceptive perceptions are heightened allowing for development of sounds for the purpose of communication. Tactile, proprioceptive, gustatory, and olfactory perceptions are integrated, allowing for primitive self-feeding. A. Six to twelve months B. Two to three years C. First six months D. Four to seven months

True

T/F: OTA notes are not required to be cosigned by an OT by the AOTA, but state and federal governments may mandate cosigning as a tangible way to demonstrate compliance wit OTA supervisory laws and regulations.

A

Erhardt Developmental Prehension Assessment is an observation checklist based on performance in three clustered areas including involuntary arm-hand patterns; voluntary movements of approach; and prewriting skills. What is the age range for the motor assessment Erhardt Developmental Prehension Assessment? A) Children of all ages (All cognitive levels, with neurodevelopmental disorders) B) Birth - 6 years (children with motor, speech-language, and/or hearing disorders) C) 4 years - 21 years D) Birth - 3.5 years

B

Excessive attention and alertness that guards against potential danger is A) Distractability B) Hypervigilance C) Selective Attention

Swallow

Facilitate ________________ by lip closure, and by placement and slight downward pressure of the spoon on the middle aspect of the tongue.

True

T/F: Observation is a major evaluation tool used during the administration process of all assessments and throughout the OT process.

True

T/F: Part A covered services have specific time limits and also require deductible and coinsurance payments by the beneficiary. Annual deductible fees must be paid by patient. Twenty percent of home health care must be paid by patient.

A

Fine pincer grasp begins at _____ ____________ A) 12 months B) 9 months C) 6 months

B

First Step Screening Test for Evaluating Preschoolers assesses cognition, communication, physical, social and emotional, and adaptive functioning. What is the age range for the developmental assessment First Step Screening Test for Evaluating Preschoolers? A) 1 month to 6 years B) 2 yrs 9 mos - 6 yrs 2 mos C) 1 to 42 mos

B

For a client with an L2 spinal cord injury, which statement BEST describes the muscle segments below the injury level 1 to 2 months postinjury? A. Senses are elevated. B. Muscles are spastic. C. Sympathetic functions are hypoactive. D. Muscles are flaccid.

dermatome pattern

For a neurological disorder, would one assess for sensation using a dermatome pattern or test distal to proximal?

Distal to proximal (following peripheral nerves)

For a peripheral nerve injury, would one assess for sensation using a dermatome pattern or test distal to proximal?

7-12 years

Games occur for children between the ages of ____ -_____ __________. Child begins to participate in cooperative peer groups with a growing interest in competition.

A

Hands come together at mideline for bilateral reaching with shoulders abducted with partial internal rotation, forearm pronation, and full finger extension at ____________. A. 4 months B. 2 months C. 12 months D. 6 months

True

T/F: Pitting edema is acute and brawny edema is chronic.

B (HELP for preschoolers is available for children without delays)

Hawaii Early Learning Profile is a non-standardized assessment that is administered in the child's natural environment and in the context of the family during typical routines. Assesses cognitive, language, gross motor, fine motor, social-emotional, and self-help. What is the age range for the developmental assessment Hawaii Early Learning Profile? (Note: this assessment is for infants and children with developmental delays, disabilities, or are at risk) A) 2 years 9 months to 5 years 8 months B) birth - 3 years C) 6 months to 7.5 years

B

Hemispheric specialization for specific tasks varies with different individuals (handedness is considered to be stable by five years, but strong preferences can be seen much earlier. This is known as _______________ A. Fine motor coordination B. Laterality C. Visual motor integration D. Bilateral integration

1, 5, 10

How many minutes after the baby is born is the apgar taken? (hint: more than once)

Every 6 months

How often are early intervention reviews submitted by all professionals to determine if services should continue?

B (The Safe Routes to School program was funded from 2005 to 2012 as part of the SAFETEA-LU legislation (Pub. L. 109-59) and provided 100% federal funding to facilitate states' initiatives to create safe environments surrounding schools and encourage children to bike and walk to school as part of developing a healthy lifestyle. In 2012 these initiatives were combined with others as part of the federal Transportation Alternatives Program.)

In 2005, Congress passed a federal transportation bill, SAFETEA-LU, which included funding for a program that encouraged children (including those with disabilities) to bike and walk to school, acted to reduce air-polluting traffic near schools, and encouraged healthy lifestyles from a young age. What was the name of this initiative? A. WalkFit to School B. Safe Routes to Schools C. Children With Disabilities Act D. Healthy American Students

D

Increased dissociation of body sides, allows for unilateral reaching with less abduction and internal rotation of the shoulder, and the hand is more open at ___________. A. 4 months B. 2 months C. 9 months D. 6 months

B

Inferior pincer grasp begins at _____ ____________ A) 12 months B) 9 months C) 6 months

C

Which frames of reference would an OT working under the medical model be most likely to use? A) Role acquisition and cognitive remediation B) Life-style performance and occupation adaptation C) Biomechanical and neurodevelopmental

Continuous (A supervisory OTA or OT must be within auditory and/or visual contact in the immediate area of the aide during the aide's task performance)

Is intermittent or continuous supervision required for an OT aide that is performing client related tasks?

C (They can increase blood pressure and heart rate, so they should be avoided.)

Isometrics are a contraction without movement and can sometimes produce a more forceful contraction. Which of the following conditions is contraindicated for isometrics? A) Hypotension and seizure disorders B) Fractures in the remodaling stage C) Hypertension and cardiovascular problems

12 months

Jaw is firm, there is rotary chewing allowing for good bite on a hard cookie

birth-2

Labyrinthine/optical (head) righting is present at _________ - ________ months and persists throughout the lifespan.

A

Medicare services provided in acute care hospitals receive a prospective, predetermined rate based on ______________. (Per case rate covers all services including OT, Is a fixed dollar amount regardless of length of stay, includes treatment supplies per case, individual hospitals determine the combination of services a patient will receive) A) DRGs (Diagnostic Reltated Groups) B) Co-insurance C) Per Diem D) Capitation

A

Miller Assessment for Preschoolers assesses sensory and motor, cognitive abilities including verbal and nonverbal, and combined abilities using a complex task index. What is the age range for the developmental assessment Miller Assessment for Preschoolers? A) 2 years 9 months to 5 years 8 months B) birth - 3 years C) 6 months to 7.5 years

A

OTR®s have an ethical responsibility, through the evaluation process, to identify impairments in occupational performance that may correlate with driving risks and to inform clients of them even if they do not have a legal responsibility to report them to the state. This obligation is consistent with which principle articulated in the Occupational Therapy Code of Ethics and Ethics Standards? A. Principle 1: Beneficence B. Principle 3: Autonomy and confidentiality C. Principle 4: Social justice D. Principle 6: Veracity

True

T/F: Skilled rehab intervention is mandatory. Delineate the specific skilled care rendered. NOtes must show therapeutic intervention: i.e. dressing does not indicate therapeutic concerns. Decreasing extensor tone to enable dressing meets these criteria.

28, Third trimester

Movement: At ______ weeks, primitive motor reflexes, rooting, suck, swallow, palmar grasp, plantar grasp, MORO, and crossed extension.

Second trimester

Movement: when does a baby begin quickening, sleep states, grasp reflex, reciprocal and symmetrical limb movements?

7 1/2, First trimester

Movement: when does a baby develop sucking, hiccuping, fetal breathing, quick generalized limb m/m, positional changes, and at ___ ____ weeks, bend neck and trunk away from perioral stroke

muscle, tissue, end plate, stretch, mature, maturing

Muscle spindle: In the first trimester m____________ starts to differentiate and t____________ becomes specialized. In the second trimester motor e______ p_________ forms, clonus response to s_____________. Third trimester: some muscles are m______________ and functional while others are still m_______________

4-6, 5

Neck righting and body righting are present at _______ - _________ months and integrates at __________ years

Coordination

O'Conner Dexterity Test, Nine-hole Peg Test, Jebsen-Taylor hand function test, Minnesota Rate of Manipulation test, Crawford Small Parts Dexterity Test, and Purdue Pegboard test are all assessments for c__________________.

C

OT in SNFs is covered if the patient requries skilled nursing or skilled rehab (i.e. OT, PT, ST) on a daily basis (min. of 5 days/week). Which form of reimbursement covers this area of therapies? A) Medicare Part B B) Medicare Part A C) Resource Utilization Groups (RUGs) D) Prospective Payment System

D (Durable medical equipment is excluded from home health agencies (HHA) PPS)

OT in home care is covered by Medicare if the person is deemed home-bound and needs intermittent skilled nursing care, PT, or ST before OT began. Which form of reimbursement covers this area of therapies? A) Medicare Part B B) Medicare Part A C) Resource Utilization Groups (RUGs) D) Prospective Payment System (PPS)

A

OT is covered as an outpatient services when provided by or under arrangements with any Medicare Certified provider (i.e. hospital, SNF, home health agency, rehab agency, a clinic) or when provided as part of a ... A) Comprehensive Rehabilitation Facility Services (CORF) B) Resource Based Relative Value Scale (RBRVS) C) Medicare Certified Provider D) Outcome and Assessment Information Set (OASIS)

True

T/F: The following assessments are used to assess pain: McGill Pain Questionnaire, Pain Disability Index, and Functional Interference Estimate.

A

OTR®s have an ethical responsibility, through the evaluation process, to identify impairments in occupational performance that may correlate with driving risks in clients that may potentially harm other road users. As such, even if they do not have a legal responsibility to report these impairments to the state, reporting because of an ethical concern should occur. This obligation is consistent with which principle articulated in the Occupational Therapy Code of Ethics and Ethics Standards? A. Principle 2: Nonmaleficence B. Principle 3: Autonomy and confidentiality C. Principle 4: Social justice D. Principle 6: Veracity

A (B, C, D: These are examples of work participation settings in which OTR®s work, but these settings do not use the place-and-train model.)

Of what is the place-and-train approach an example? A. Supported employment B. Transitional employment C. Job matching D. The clubhouse model

Level II

On the Ranchos Los Amigos Scale a person who has a generalized response and requires total assistance is considered a ____________ _____. -General response to stimuli -repeated auditory stimuli with increased/decreased activity -external stimuli - gross body m/m -response may be significantly delayed

Level III

On the Ranchos Los Amigos Scale a person who has a localized response and requires total assistance is considered a ____________ ______. -withdrawal or vocalization to pain -toward/away from auditory stim -blinks to strong light in visual field -discomfort by pulling tubes/restraints -responds inconsistently to simple commands -Responses directly related to type of stimulus -May respond to friends/family but not others

Level I

On the Ranchos Los Amigos Scale a person who has no response and requires total assistance is considered a ___________ ____.

Level VII

On the Ranchos Los Amigos Scale a person who is automatic, appropriate and requires minimal assistance for Daily Living Skills is considered a ____________ ______ -Consistently oriented -attends to highly familiar tasks in non-distracting environment w min redirection -Min supervision for new learning -Carry over of new learning -Initiates and carries out steps to complete familiar activities, but has shallow recall of what he/she has been doing -Able to monitor accuracy and completeness of each step in ADLs and IADLs w/ min A -Superficial awareness of condition, still a safety concern -Min supervision for safety in routine home and community activities -Unrealistic planning of future -unaware of consequences -overestimates abilities -unaware of others' needs/feelings -oppositional/uncooperative -unable to recognize inappropriate social interaction behavior

Level VI

On the Ranchos Los Amigos Scale a person who is confused, appropriate and requires moderate assistance is considered a ____________ ______ -inconsistently oriented -attends to highly familiar tasks in non-distracting environment w moderate redirection -remote memory = more detail than recent -vague recognition of staff -Use assistive memory aide with max A -emerging awareness of response to family, self, and needs -Mod A for problem solving for task completion -Supervised for old learning -carry over for relearned familiar tasks -Max A for new learning w/ little carry over -Unaware of impairments, disabilities, safety -Consistently follows simple directions -Verbal expressions are appropriate

Level V

On the Ranchos Los Amigos Scale a person who is confused, inappropriate non-agitated and requires maximal assistance is considered a ____________ ______ -Alert -Not oriented to person, place, time -frequent brief periods of non-purposeful attention -Severely impaired recent memory -often demonstrates inappropriate use of objects w/o external direction -May be able to learn previously learned tasks with verbal cues -Unable to learn new info -Responds appropriately to simple commands with external structures and cues -Converses on a social, automatic level for brief periods of time -Verbalizations are inappropriate when external stimuli are not provided

Level IV

On the Ranchos Los Amigos Scale a person who is confused/agitated and requires maximal assistance is considered a ____________ ______ -Alert -purposeful attempts to remove IVs/restraints and crawl out of bed -May perform motor activities - sitting, reaching, walking w/o apparent purpose -brief, non-purposeful moments of sustained alternatives and divided attention -may cry out or scream out of proportion to stimulus -mood swings -unable to cooperate with tx efforts -verbalizations are frequently incoherent and/or inappropriate to act. or env.

Level X

On the Ranchos Los Amigos Scale a person who is purposeful, appropriate and modified independent for Daily Living Skills is considered a ____________ ______ -Able to handle multiple tasks simultaneously in all environments - may require periodic breaks -ind. procures, creates, and maintains own memory assistive devices -ind. initiates familiar and unfamiliar tasks, may require extra time -Anticipates impact of impairments and problems before they occur - may require extra time for compensatory strategies. -Accurately estimates abilities - makes adjustments ind. -automatically recognizes needs of others, responds appropriately -Periodic periods of depression may occur -Irritability and low frustration tolerance when sick, fatigued, and/or under emotional distress -Social interaction behavior is consistently appropriate.

Level VIII

On the Ranchos Los Amigos Scale a person who is purposeful, appropriate and requires standby assistance for Daily Living Skills is considered a ____________ ______ -Independently attends to and completes familiar tasks for 1 hour in distracting env. -recalls past and integrates recent events -Uses assistive memory devices for daily schedule -Initiates and carries out steps to complete familiar activities, with SBA and can modify activities with Min A -No A when task is learned -Aware of and acknowledges impairments and disabilities when they interfere with task completion, requires SBA to take appropriate corrective action -Thinks about consequences w/ min A -over/underestimates abilities -Acknowledges others' needs/feelings with Min A -Depressed -Irritable -Low frustration tolerance/easily angered -Argumentative -Self-centered -Uncharacteristically ind./dep -recognizes/acknowledges inappropriate social interactions while it's occurring and takes corrective action with Min A.

Level IX

On the Ranchos Los Amigos Scale a person who is purposeful, appropriate and requires standby assistance for Daily Living Skills is considered a ____________ ______ -Shifts back and forth bw tasks and completes them accurately for at least 2 hours - Uses assistive memory devices for daily schedule -Initiates and carries out steps to complete familiar activities ind. and can modify activities with assistance -Aware of and acknowledges impairments and disabilities when they interfere with task completion, takes appropriate corrective action, but requires SBA to anticipate problems before they occur -Able to think about consequences when with assistance when requested -Accurately estimates abilities, but requires SBA to adjust task demands -Acknowledges others' needs and feelings and responds appropriately with SBA -Depression may continue -May be easily irritable -May have low frustration tolerance -Able to self monitor appropriateness of social interaction with SBA

B

PLISSIT (Permission, limited information, specific suggestions, intensive therapy) model is a guide for appropriate ____________ ____________ A. Play interventions B. Sexual interventions C. Work interventions D. Self-care interventions

37, 4-6

Palmar grasp is present at _________ weeks and integrates at ________-________ months

dyskinesia

Parkinson's: during early treatment, side effects from carbidopa-levodopa are not a major problem. As the diseases progresses some people may experience involuntary movements known as __________________, primarily when the medication is having its peak effects.

C

Pediatric Evaluation of Disability Inventory (PEDI) is a standardized behavior checklist and rating scale that assesses capabilities and detects functional deficits in self-care, mobility, and social skills. What is the age range for the developmental assessment Pediatric Evaluation of Disability Inventory. A) 2 years 9 months to 5 years 8 months B) birth - 3 years C) 6 months to 7.5 years

B

Prone Position: At which age does a child begin airplane posturing in prone position; chest and thighs lift off surface? A) 5-6 months B) 5-8 months C) 0-2 months D) 9 months

B

Prone Position: At which age does a child pivot in prone position and move from prone to sit? A) 9 months B) 8-7 months C) 5-6 months D) 0-2 months

Medicare

Which health insurance plan is the largest single payer for OT services?

28, 9

Plantar grasp is present at ___________ weeks and integrates at ___________ months

C

Play History assesses behavior and play opportunities. The primary caregiver provides information on the child including general info., previous play experience, and actual play that occurs over three days of play. What is the age range for Play History? A) 0-6 years B) Infancy to 6 years C) Children and Adolescents D) 15 months - 10 years

D

Preschool Visual Motor Integration Assessment evaluates visual motor integration and visual perceptual skills including perception in space, awareness of spatial relationships, color and space discrimination, matching two attributes simultaneously and the ability to reproduce what is seen and interpreted. What is the age range for the visual motor and visual perception assessment Preschool Visual Motor Integration Assessment (PVMIA)? A) birth - 6 months B) 2 - 100 years C) 4-10 years, 11-74 D) 3.5-5.5

D

Principle 2, Nonmaleficence, of the Occupational Therapy Code of Ethics (2015) imparts an obligation to do no harm to others. Which of the following ethical violations is MOST likely a violation of the principle of Nonmaleficence? A. The director of an occupational therapy program receives a gift from a vendor in appreciation for the purchase of several items for the program. Because the gift is worth less than $75, the director decides not to report it to the employer. B. A clinical supervisor offers a reduced fee for hand management services to a physician who regularly refers clients to the occupational therapy program. C. To convince a research participant to continue in a low-risk study, an occupational therapy researcher offers an additional $50 incentive to the participant. D. An OTR® meets the sibling of a service recipient and has an intimate relationship with the sibling while continuing to provide occupational therapy for the family member.

C

Raking begins at ______ ___________ A) 12 months B) 9 months C) 6 months

B

Release: At which age does a child complete one-stage transfers; taking hand and releasing hand perform actions simultaneously? A) 7-9 months B) 6-7 months C) 7-10 months D) 8 months

D

Release: At which age does a child complete two-stage transfers; taking had grasps before releasing hand lets go? A) 4-8 months B) 1-4 months C) 4 months D) 5-6 months

D

Release: At which age does a child demonstrate clumsy release into small container; hand rests on edge of container? A) 7-10 months B) 6-7 months C) 9-10 months D) 10-14 months

A

Release: At which age does a child demonstrate precise controlled release into small container with wrist extended? A) 12-15 months B) 6-7 months C) 9-10 months D) 10-14 months

A

Release: At which age does a child demonstrate volitional release? A) 7-9 months B) 6-7 months C) 7-10 months D) 8 months

B (at 4 months mutual fingering in midline occurs)

Release: At which age does a child display involuntary release? A) 4-8 months B) 1-4 months C) 4 months D) 5-6 months

C

Release: At which age does a child press down on surface to release? A) 7-9 months B) 6-7 months C) 7-10 months D) 8 months

D

Release: At which age does a child release above a surface with wrist flexion? A) 7-9 months B) 6-7 months C) 7-10 months D) 8 months

C

Release: At which age does a child release into a container with wrist straight? A) 7-10 months B) 6-7 months C) 9-10 months D) 10-14 months

A

Release: At which age does a child transfer objects from hand to hand? A) 4-8 months B) 1-4 months C) 4 months D) 5-6 months

D

Rolling: At which age does a child roll from prone to side accidentally due to poor control of weight shift, rolls from supine to side. A) 5-6 months B) 6-14 months C) 8-7 months D) 3-4 months

A

Rolling: At which age does a child roll from prone to supine, supine to side with right and left leg performing independent mm, rolls from supine to prone with right and left leg performing independent mm? A) 5-6 months B) 6-14 months C) 8-7 months D) 3-4 months

B

Rolling: At which age does a child roll segmentally with roll initiated by the head, shoulder, or hips? A) 5-6 months B) 6-14 months C) 8-7 months D) 3-4 months

28, 3

Rooting is present at ____________ weeks and integrates at _________ months.

state regulatory boards

S_________________ R______________ B_______________: public bodies created by state legislatures to assure the health and safety of the citizens of that state. They protect the public from potential harm that might be caused by incompetent or unqualified practitioners.

A

Sensory Processing Measure (SPM) measures sensory processing, praxis, and social participation across different environments. Assesses visual, auditory, tactile, olfactory-gustatory, proprioceptive, and vestibular behaviors. Home form completed by primary caregiver, and school form completed by teacher. What is the age range for this assessment? A) Elementary aged children B) 3-5 years C) 4-8.11 years D) 1-18 months of age

A

Sensory Profile: Adolescent/Adult allows clients to identify their personal behavioral responses and develop strategies for enhanced participation. What is the age range for Sensory profile: adolescent/adult sensory profile? A) 11 - 65 B) birth to 36 months

No

Should a person with circulation problems place their extremity above their head to assist with edema management?

Sphygomomanometer cuff or vigorometer/bulb dynamometer

Should a standard dynamometer or a Sphygomomanometer cuff or vigorometer/bulb dynamometer be used for persons with arthritis?

D

Sitting: At which age can a child be held in sitting? Head bobs, back is rounded, hips are apart, turned out, and bend; head is steady; chin tucks; able to gaze at floor. A) 5-6 months B) 5-10 months C) 7-8 months D) 0-3 months

D

Sitting: At which age can a child get to sitting from prone? A) 5-6 months B) 5-10 months C) 7-8 months D) 6-11 months

C

Sitting: At which age can a child rotate upper body while lower body remains stationary, equilibrium reactions and protective responses present? A) 5-6 months B) 5-10 months C) 7-8 months D) 6-11 months

B

Sitting: At which age can a child sit alone? Sits alone steadily, initially with wide base of support; able to play with toys in sitting position? A) 5-6 months B) 5-10 months C) 7-8 months D) 0-3 months

A

Sitting: At which age can a child support self in sitting? Sits alone momentarily; increased extension in back; sits by propping forward on arms; wide base; legs bent; periodic use of high guard position; protectice response when falling to front? A) 5-6 months B) 5-10 months C) 7-8 months D) 0-3 months

D

Sitting: At which age does a child exhibit active protective extension backwards; first with bent elbows, then straight elbows; able to move in and out of sitting into other positions? A) 8-10 months B) 11-12 months C) 9-18 months D) 10-12 months

B

Sitting: At which age does a child exhibit trunk control and equilibrium responses are fully developed in sitting position; further increase in variety of positions possible? A) 8-10 months B) 11-12 months C) 9-18 months D) 10-12 months

C

Sitting: At which age does a child rise from supine by first rolling over to stomach then pushing up into four-point position? A) 8-10 months B) 11-12 months C) 9-18 months D) 10-12 months

C

Sitting: At which age does a child rise from supine by first rolling to side then pushing up into sitting position? A) 8-10 months B) 11-12 months C) 11-24 months D) 10-12 months

A

Sitting: At which age does a child sit well without support; lefts are closer; full upright position; knees straight; increased variety of sitting positions; difficult fine motor tasks may require wider base of support? A) 8-10 months B) 11-12 months C) 9-18 months D) 10-12 months

C

Some of the adults in an inpatient psychiatric hospital participate in group activities in which they perform a shared, short-term activity with another member in a comfortable, cooperative manner. The focus of the group is to develop interactions beyond those that the activity requires and enable members to give and seek assistance. Suitable activities include simulated, clearnly defined, structured activities which enable members to practice and learn needed skills, attitudes, and knowledge within the group. Which group model does this follow? A. Parallel B. Mature C. Project D. Cooperative

True

T/F: Under IDEA, IEP;s annual goals include academic functional goals, enables services to be provided to students as soon as learning needs become apparent via a Response to Intervention (RtI) approach. (RtI provides evidence based early intervention services to children who are having difficulty learning to prevent academic failure).

28, 3-5

Suck-swallow is present at _________ weeks and integrates at ______ - _____ months.

c

Supine Position: At which age does a child independently lift head and bring feet to mouth, bring hands to feet, reach for a toy with one or both hands, and hands are predominantly open? A) 9 months B) 8-7 months C) 5-6 months D) 0-2 months

4-6, 8-12

Symmetric tonic neck is present at _______-_______ months and integrates at _______ - _______ months.

True

T/F: A child who is removed from school due to disciplinary action, must still be received by the child even if he/she is removed to an alternative placement.

True

T/F: A fundamental technique in the dynamic interactional approach is awareness questioning to help the indiv. identify success, detect errors, estimate task difficulty and predict outcomes.

True

T/F: Active ROM should be performed with PROM is greater than AROM.

True

T/F: DBT = dialectical behavior therapy, addresses suicidal thoughts and actions and self injurious behaviors. Commonly used with individuals with boarderline personality disorder. Also used to treat depression, substance abuse, and/or eating disorders. TEACHES ASSERTIVENESS, COPING AND INTERPERSONAL SKILLS. DBT GROUPS ADDRESS HOW THE ACQUISITION OF SKILL AFFECTS OCCUPATIONAL PERFORMANCE AND PROVIDE OPPORTUNITIES TO PRACTICE NEW SKILLS.

True

T/F: For patients with CNS dysfunction, a proper wheelchair prescription involves firm seats to provide stability and a solid base for seating systems that can be used to prevent decubiti, contractures, and deformities, and to increase sitting tolerance, proper positioning and functional abilities.

True

T/F: Intracranial pressure can possibly lead to paralysis of the sixth cranial nerve resulting in visual impairments.

True

T/F: Manual edema mobilization and retrograde massage are contraindicated when cardiac edema is present.

True

T/F: Medicare does not cover most chronic illnesses, long term supportive care, or all medical expenses incurred when ill.

D

Test of Playfulness assesses a child's playfulness based on observations according to four aspects of play: intrinsic motivation, internal control, disengagement from constraints of reality, and framing. What is the age range for Test of Playfulness (ToP)? A) 0-6 years B) Infancy to 6 years C) Children and Adolescents D) 15 months - 10 years

D

Test of Sensory Functions in Infants is a standarized test that assesses the level of an infant's sensory responsiveness to a variety of sensory stimuli. What is the age range for this assessment? A) Elementary aged children B) 3-5 years C) 4-8.11 years D) 1-18 months of age

C

Test of Visual Motor Skills assesses eye hand coordination for copying geometric designs. What is the age range for the visual motor and visual perception assessment Test of Visual Motor Skills (TVMS) and (TVMS-UL) upper level? A) 4-19 B) All ages C) 2-13, 12-40 D) 4-95

A

Test of Visual Perceptual Skills assesses visual-perceptual skills and differentiates these from motor dysfunction. Seven visual perceptual skills including visual discrimination, visual memory, visual spatial relationships, visual form constancy, visual sequential memory, visual figure-ground, and visual closure. What is the age range for the visual motor and visual perception assessment Test of Visual-Perceptual Skills, 3rd ed. (TBPS3) A) 4-19 B) All ages C) 2-13, 12-40 D) 4-95

C

The BOT-2 assesses fine motor precision, fine motor integration, manual dexterity, bilateral coordination, balance, running speed and agility, upper limb coordination and strength (hand and foot preference). The assessment considers speed, duration, and accuracy of performance. What is the age range for the motor assessment BOT-2? A) Children of all ages (All cognitive levels, with neurodevelopmental disorders) B) Birth - 6 years (children with motor, speech-language, and/or hearing disorders) C) 4 years - 21 years D) Birth - 3.5 years

C

The Bayley Scales of Infant Development, 3rd edition assesses cognitive, language, and motor (perfmormace based skills), and social emotional and adaptive skills. What is the age range for the developmental assessment A) 1 month to 6 years B) 2 yrs 9 mos - 6 yrs 2 mos C) 1 to 42 mos

C

The Developmental Test of Visual Perception (2nd Edition) and Developmental Test of Visual Perception - Adolescent and Adult (DTVP-A) assesses visual perceptual skills and visual motor integration for levels of performance and for disgning interventions and monitoring processes. The DVPT-2 is comprised of 8 subtests: eye-hand coordination, copying, spatial relations, visual-motor speed, position in space, figure ground, visual closure, form consistency. DVPT-A is comprised of 4 subtests: visual-motor integration, composite index, motor-reduced visual perception composite index. What is the age range for the visual motor and visual perception assessment Developmental Test of Visual Perception DTVP-2 and DTBP-A? A) birth - 6 months B) 2 - 100 years C) 4-10 years, 11-74 D) 3.5-5.5

A (6 month level is considered the norm, the EDVA-can be used for assessing older children)

The Erhardt Developmental Vision Assessment is a behavior rating scale to determine visuo-motor development that assesses involuntary visual patterns including eyelid reflexes, pupillary reactions, doll's eye reponses, and voluntary patterns including fixation, localization, ocular pursuit, and gaze shift. What is the age range for the visual motor and visual perception assessment Erhardt Developmental Vision Assessment (EDVA)? A) birth - 6 months B) 2 - 100 years C) 4-10 years, 11-74 D) 3.5-5.5

D (For children aged 4-10 items 1-40 are administered; persons aged 10+ years items 14-65 are administered)

The Motor-Free Visual Perception Test evaluates visual perception in five areas including spatial relationships, visual discrimination, figure ground, visual closure, and visual memory. What is the age range for the visual motor and visual perception assessment Motor-Free Visual Perception Test (MVPT-3)? A) 4-19 B) All ages C) 2-13, 12-40 D) 4-95

B (Children and adults with visual field cuts or without visual impairments)

The Motor-Free Visual Perception Test- Vertical evaluates individuals with spatial deficits due to hemi-field visual neglect or abnormal visual saccades. 36 items vertically places to assess spatial relationships, visual discrimination, figure ground, visual closure, and visual memory. What is the age range for the visual motor and visual perception assessment Motor-Free Visual Perception Test Vertical (MVPT-V)? A) 4-19 B) All ages C) 2-13, 12-40 D) 4-95

B

The Peabody Developmental Motor Scales-2 assesses gross and fine motor development including reflexes, sustained control, locomotion, object manipulation, grasping, and visual motor integration. What is the age range for the motor assessment for PDMS-2? A) Children of all ages (All cognitive levels, with neurodevelopmental disorders) B) Birth - 6 years (children with motor, speech-language, and/or hearing disorders) C) 4 years - 21 years D) Birth - 3.5 years

B

The School Function Assessment evaluates and monitors functional performance in order to promote participation in a school environment. What is the age range for this assessment? A) 15 years + (with physical disabilities) B) School aged children

C

The Sensory Integration and Praxis Test (SIPT) is a standarized test that addresses the relationship among tactile processing, vestibular-proprioceptive processing, visual perception, and practic ability. This assessment requires certification! What is the age range for this assessment? A) Elementary aged children B) 3-5 years C) 4-8.11 years D) 1-18 months of age

B

The Sensory Profile: Infant/toddler assesses reactions to daily sensory experiences. Obtains caregiver's judgment and observation of a child's sensory processing modulation, and behavioral and emotional responses in each sensory system via a caregiver questionnaire. What is the age range for the Sensory Profile: Infant/toddler sensory profile? A) 11 - 65 B) birth to 36 months

B

The Transdisciplinary Play-Based Assessment measures a child's development, learning style, interaction patterns, and behaviors to determine need for services. Employs team observations based on six phases: developmental domains of cognitive, social-emotional, communication and language, and sensorimotor. What is the age range for Transdisciplinary Play-Based Assessment? A) 0-6 years B) Infancy to 6 years C) Children and Adolescents D) 15 months - 10 years

A (Donepezil is a cholinergic-modulating drug that may improve memory and cognition and reduce negative mood, anxiety, and hallucinations; however, dizziness is a side effect.)

The caregiver of a client with Alzheimer's disease questions the home health OTR® about a recent increase in the client's dosage of donepezil (Aricept) prescribed by the physician. The caregiver is particularly concerned about potential consequences of the increased dosage. What is the OTR®'s most appropriate suggestion for the caregiver? A. Observe the client for signs of dizziness, which increases potential for falls. B. Because the medication can cause photosensitivity, keep the client out of direct sunlight. C. The physician will probably monitor the client closely for potential drug toxicity. IncorrectD. Monitor the client's hydration carefully because the drug can cause dry mouth and constipation.

B

The onset of Backward Parachute (protective extension backward) (quickly but firmly tip infant off balance backward) is at _______________, and _______________. Response: Backward arm extension or arm extension to one side spinal rotation. Protects body to prevent a fall; unilaterally facilitates. A. 9-10 months, integrates at 18 months B. 9-10 months, persists C. 6 months, integrates at 18 months D. 6 months, persists

C

The onset of Downward Parachute (protective extension downward) (rapidly lower infant toward supporting surface while suspended vertically) is at __________, and _____________. Response: extension of lower extremities. Allows accurate placement of lower extremities. A. 7 months, persists B. 4 months, integrates at 12 months C 4 months, persists D. 7 months, integrates at 12 months

B

The descriptive categories established by Center for Medicare/Medicaid Services (CMS) that determine the level of payment at a per case rate is A) Fee for service B) Diagnostic related groups (DRGs) C) Health insurance marketplace D) Health maintenance organization

True

The ethics commission is responsible for informing and educating members about current ethical issues, upholding practice and education standards of the profession, monitoring the behavior of members, and reviewing allegations of unethical conduct.

A

The executive branch of an accounting firm and several employees work together to develop a better work environment for all staff while ensuring excellent customer service. An occupational therapist participates in the group as a peer in order to enable members to reinforce behaviors which result in need satisfaction and task completion. Which group model does this follow? A. Mature B. Cooperative C. Parallel D. Project

Sensory receptors

The following ______________ ______________ of the hand include the Pacinian corpuscles - responsible for vibration, Ruffini end organs - responsible for tension, and Merkel cells - responsible for pressure

Motor Assessments

The following are ______________ _______________: -Bruininks-Oseretsky Test of Motor Proficiency, 2nd edition (BOT-2) -Erhardt Developmental Prehension Assessment (EDPA), (EDPA-S) Short form -Peabody Developmental Motor Scales (PDMS-2) -Toddler and Infant Motor Evaluation

Developmental Assessments of Neonates

The following are ________________ _________________ ____ _____________________: -Assessment of Preterm Infants' Behavior (APIB) - assesses behavioral org in response to increasing sensory and environmental stim. -Neurological Assessment of Pre-term and Full-term Newborn Infant (NAPFI) - Rating scale consisting of brief neurological examination incorporated into routine assessment.

Psychological and Cognitive Assessments

The following are _________________ ______ _________________ ___________________: -Childhood Autism Rating Scale (CARS) -Coping Inventory and Early Coping Inventory

Sensory Processing Assessments

The following are _________________ _______________ _______________: -Sensory Profile: Infant/toddler sensory profile -Sensory profile: adolescent/adult sensory profile

Social Participation Assessments

The following are _________________ ________________ ______________________: -Participation Scale (P Scale) (Version 6.0) -School Function Assessment (SFA)

Development Assessments

The following are ___________________ ________________: -Denver Developmental Screening Test II -Bayley Scales of Infant Development, 3rd Edition (BSID-III) -First Step Screening Test for Evaluating Preschoolers -Hawaii Early Learning Profile, Revised (HELP) -Miller Assessment for Preschoolers -Pediatric Evaluation of Disability Inventory (PEDI)

Play Assessments

The following are ____________________ ___________________: -Play History -Revised Knox Preschool Play Scale (RKPPS) -Test of Playfulness (ToP) -Transdisciplinary Play-Based Assessment (TPBA)

Visual Motor and Visual Perception Assessments

The following are v___________ m__________ ______ v____________ p_____________ _______________: -Beery-Buktenica Developmental Test of Visual Motor Integration - VMI, 6th edition -Developmental Test of Visual Perception, 2nd edition (DTBP-2) (DTBP-A) adolescent and adult -Erhardt Developmental Vision Assessment (EDVA) (EDVA-S) Short -Preschool Visual Motor Integration Assessment (PVMIA) -Motor Free Visual Perception Test (MVPT-3) -Motor Free Visual Perception Test-Vertical (MVPT-V) -Test of Visual - Motor Skills (TVMS) and Test of Visual-Motor Skills: Upper Level (TVMS-UL) -Test of Visual-Perceptual Skills, 3rd Ed. (TVPS3)

A

The inability to perform rapidly alternating movements is A) Adiadochokinesia B) Agnosia C) Ataxia D) Distractability

D

The need for a satisfying balance between work/productive activities, leisure/play, and rest (e.g. forced leisure due to involuntary unemployment requires intervention). This is an example of: A. Psychophysical B. Love and acceptance C. Mastery D. Temporal balance and regularity

D (Joint protection principles are ideally taught early in the disease process to decrease joint stress and damage.)

The occupational profile of a client recently diagnosed with bilateral osteoarthritis of the first carpometacarpal joints of the thumbs reveals that the client is retired and enjoys reading, playing cards with friends, and painting. The client has insurance coverage for only three occupational therapy sessions. What client education topics should the OTR® focus on FIRST? A. Hand strengthening exercises the client can perform independently B. Orthoses that may be fabricated to support and protect affected joints C. Thermal modalities to reduce pain and stiffness D. Joint protection principles

2.8 (grabbing)

Which Allen Cognitive level is indicative of 24-hour nursing care to stabilize grab bars, rails, furniture, point out stairs, edge of bathtub, provide food, and bathe.

A

The onset of Forward Parachute (protective extension forward) (suddenly tip infant forward toward supporting surface while vertically suspended) is at ______________, and _______________. Response: sudden extension of upper extremities, hand opening, and neck extension. Allows accurate placement of upper extremities in anticipation of supporting surface to prevent a fall. A. 6-9 months, persists B. 5 months, integrates at 2 years C. 6-9 months, integrates at 2 years D. 5 months, persists

B

The onset of Galant (hold infant in prone suspension, gently scratch or tap alongside the spine with finger, from shoulders to buttocks) is at _____________, and integrates at _____________. Facilitates lateral trunk movements necessary for trunk stabilization. A. 28 weeks, 5-7 months B. 32 weeks, 2 months C. 32 weeks, 5-7 months D. 28 weeks, 2 months

B

The onset of Moro (rapidly drop infant's head backward) is at _____________, and integrates at __________. Facilitates the ability to depart from dominant flexor posture: protective response. Two phases: 1. arm extension/abduction hand opening; 2. arm flexion and adduction. A. Birth, persists B. 28 weeks, 4-6 months C. Birth, 4-6 months D. 28 weeks, persists

C

The onset of Prone Tilting (After positioning infant in prone, slowly raise one side of the supporting surface) is at ________________, and ________________. Response: curving of the spine toward the raised side (opposite to the pull of gravity); abduction/extension of arms and legs. Maintain equilibrium without arm support; facilitate postural adjustments in all positions. A. 8 months, integrates 24 months B. 8 months, persists C. 5 months, persists D. 5 months, integrates 24 months

A

The onset of Quadruped tilting (After positioning infant on all fours, slowly raise one side of the supporting surface) is at ___________________, and ___________________. Response: curving of the spine toward the raised side (opposite to the pull of gravity); abduction/extension of arms and legs. Maintain equilibrium without arm support; facilitate postural adjustments in all positions. A. 9-12 months, persists B. 9-12 months, integrates 3 years C. 5 months, persists D. 5 months, integrates 3 years

C

The onset of SUCK-SWALLOW and TRACTION are at _________, and integrate at _______ - ________. A. 32 weeks, 5-7 months B. 32 weeks, 2-5 months C. 28 weeks, 2-5 months D. 28 weeks, 5-7 months

D

The onset of Sideward parachute (protective extension sideward) (Quickly burt firmly tip infant off-balance to the side while in sitting position) is at _____________, and _____________. Response: Arm extension and abduction to the side. Protects body to prevent a fall; supports body for unilateral use of opposite arm. A. 12 months, integrates at 5 years B. 12 months, persists C. 7 months, integrates at 5 years D. 7 months, persists

C

The onset of Standing tilting (After positioning infant in standing, slowly raise one side of the supporting surface. Response: curving of the spine toward the raised side (opposite to the pull of gravity); abduction/extension of arms and legs. Maintain equilibrium without arm support; facilitate postural adjustments in all positions. A. 12-21 months, integrates 4 years B. 24 months, persists C. 12-21 months, persists D. 24 months, integrates 4 years

B

The onset of Supine tilting and sitting tilting (after positioning infant in supine or sitting, slowly raise one side of the supporting surface) is at ______________, and _______________. Response: curving of the spine toward the raised side (opposite to the pull of gravity); abduction/extension of arms and legs. Maintain equilibrium without arm support; facilitate postural adjustments in all positions. A. 12 months, integrates 18 months B. 7-8 months, persists C. 12 months, persists D. 7-8 months, integrates 18 months

D

The participants of an inpatient treatment program for adolescents with oppositional defiant disorder work a small group that is focused on completing the same task individually. They have minimal interaction with the other members of the group, and the goal is to develop basic level of awareness, trust, and comfort with others in a group. Which group model does this follow? A. Project B. Ego-centric Cooperative C. Mature D. Parallel

A

The participation scale is designed to measure restrictions in participation related to community mobility, access to work, recreation, and social interactionw ith family, peers, neighbors, etc. What is the age range for Participation Scale (P Scale) (Version 6.0)? A) 15 years + (with physical disabilities) B) School aged children

C

The state of restlessness characterized by urgent need for movement, usually as a side-effect of meds is A) Ataxia B) Hyperactivity C) Akathisia D) Echopraxia

Olfactory, nasal

This sensorimotor development does not occur until the third trimester ___________________, n__________ plugs disappear, some __?___ perception

D

Toddler and Infant Motoro Evaluation assesses the quality of movment in five primary subtests: mobility, stability, motor organization, socila/emotional abilities, and functional performance. What is the age range for the motor assessment for Toddler and Infant Motor Evaluation? A) Children of all ages (All cognitive levels, with neurodevelopmental disorders) B) Birth - 6 years (children with motor, speech-language, and/or hearing disorders) C) 4 years - 21 years D) Birth - 3.5 years

37, 6

Tonic labyrinthine-supine and tonic labyrinthine-prone are present at >__________ weeks and integrates at _________ months

C

Transportation researchers advocate for the development of services that can foster coordination and collaboration among transportation providers both public and private and acknowledge that specially trained OTR®s can fulfill this role. What is this concept called? A. Mobility advocacy B. Mobility discovery C. Mobility management D. Mobility independence

Remediation

Using practice of scanning activities is a transfer of training approach which assumes that __________________ of the cause of the presenting symptom will result in increased functional skills.

Second trimester

Vision: in which trimester are babies startled to light, and visual processing occurs?

First trimester

Vision: in which trimester are the eylids fused and the optic nerve and cup are being formed?

Third trimester

Vision: in which trimester does fixation occur and babies are able to focus (fixed focal length)?

Percussion

WHat is the name of the finger tapping technique used to at the point of pain is used to desensitize a residual limb?

3.4 (Sustaining Actions on Objects)

Which Allen Cognitive level is indicative of Sustaining Actions on Objects: Close supervision to place objects needed to do activities of daily living in front of the person and sequence through the necessary steps to achieve acceptable results. One caregiver can supervise three persons at a time.

eye, verbal, motor

What are the three responses assessed by the Glascow Coma Scale?

C (When used in middle-stage dementia, door alarms can be useful in improving safety of the client with dementia and reducing caregiver burden.)

What assistive technology would be appropriate to recommend to a client who has middle-stage dementia or the client's caregivers? A. Medication dispensers B. Medication reminder boxes C. Door alarms D. Electric hospital bed

Vestibular system

What functions at the end of the first trimester, but is not completely developed?

D

What is the FIRST step in the process of designing a group protocol for clients with mental illness? A. Find appropriate dates and times for conducting the group. B. Identify the appropriate outcomes of the group and methods for tracking and recording them. C. Develop group goals and determine the size of the group. D. Identify a problem and the factors that will motivate the people in the group to change.

10

What is the highest apgar score for a newborn baby?

15

What is the highest score one can attain on the GLASGOW Coma Scale?

3

What is the lowest score one can attain on the GLASGOW Coma Scale?

Complex Regional Pain Syndrome (CRPS)

What is the most severe complication of a hand fracture?

A (Engagement in activity-based interventions, along with daily structure, has been documented to assist in fall prevention.)

What strategy might an OTR® appropriately recommend to caregivers of a client with Alzheimer's disease to assist with fall prevention? A. Engage the client in daily, structured activity. B. Provide visual reminders in the home environment. C. Install grab bars in the bathroom. D. Engage the client in an exercise program.

First trimester

When do taste buds occur?

0-2 years

When does exploratory play develop? Child engages in play experiences through which the child develops body scheme.

9 months

When does lateral tongue m/m make mastication of soft and mashed food effective, able to to drink from a cup; jaw is not firm

D

When does manipulation skills emerge? A. 10 months B. 6-8 months C. 12-18 months D. 18-24 months

6 months

When does strong up and down movement of the tongue occur?

A

When does the ability to use two different hands for two very different functions emerge? A. 2.5 years B. 18-24 months C. 3.5 years D. 5 years

C (By 9 months, release by full arm extension)

When does voluntary release occur? A. 1-4 months B. 6-7 months C. 7-9 months D. 5-6 months

A (When the right side of the heart fails, blood flows back into the venous system. Symptoms of right-sided heart failure include cyanotic nail beds, jaundice, and lower-extremity edema.) (B. When the left side of the heart fails, blood flows back into the lungs, causing difficulty breathing, anxiety, and cerebral hypoxia.) (C, D: Aortic stenosis and aortic insufficiency have symptoms similar to those of left-sided heart failure.)

When evaluating a client in a skilled nursing facility, the OTR® notes that the client's skin has a yellowish cast, the fingernail beds are bluish in color, and the client has noticeable edema in both lower extremities. What condition would MOST likely cause these symptoms? A. Right-sided congestive heart failure B. Left-sided congestive heart failure C. Aortic stenosis D. Aortic insufficiency

Vascular

When evaluating the UE, observation of color, trophic changes, pulse, skin temp, and Allen's test are all used to measure v_________________.

A

When facilitating lip closure, a therapist should A) Apply slight upward pressure of index finger under the child's lip B) Apply slight downward pressure of index finger above the child's lip C) Apply slight pressure above and under the child's lip D) Manually hold lips together.

A (Distributing body weight along the sitting surface (along the entire length of the thigh to just behind the knee) helps to prevent pressure sores on the buttocks and the lower back and to attain optimal muscle tone normalization to assist in prevention of pressure sores throughout the body.)

When fitting a client to a wheelchair, why is it important to ensure that the wheelchair has appropriate seat depth? A. To distribute body weight along the entire sitting surface B. To maintain the thighs in a position parallel to the floor C. To keep the depth of the chair as small as possible D. To provide support and alignment for the upper extremities

Third trimester

When in the womb can a baby respond to different tastes(sweet, sour, bitter, salt)

D

When performing a manual muscle test (MMT) on a client, the OTR® should use standard principles to guide the evaluation of muscle strength. Which statement accurately reflects those principles? A. The OTR can safely perform MMT on a client in acute pain. B. The OTR should apply the same amount of resistance for all muscles tested. C. The OTR should stabilize distally from the joint and apply resistance proximally to the joint. D. The OTR should apply resistance applied gradually and increase it as the client responds.

3.2 (Distinguishing Objects)

Which Allen Cognitive level is indicative of 24-hour nursing care to place objects needed to do the activities of daily living in front of person and complete motions for an acceptable

2.6 (Directed walking)

Which Allen Cognitive level is indicative of 24-hour nursing care to restrict walking to even surfaces in safe locations such as a room, building, or yard

4.4 (Completing goal)

Which Allen Cognitive level is indicative of completing goal: Person may live with someone who does a daily check on the environment and removes any safety hazards and solves any new problems. Person may be left alone for part of the day with procedure for obtaining help by phone or from a neighbor. Person may manage a daily allowance and go to familiar places in the neighborhood.

5.6 (Considering Social Standards of Context)

Which Allen Cognitive level is indicative of considering social standards of context: Person may respond to supervision that identifies hazards occurring as secondary effects of their actions. Person may be relied upon to follow safety precautions consistently.

5.8 (consulting with Others)

Which Allen Cognitive level is indicative of consulting with others: Person may benefit from assistance in planning for the future. Person may benefit from discussion of complications such as fatigue, joint protections, functional positioning, etc.

5.0 (Continuous Neuromuscular Adjustments)

Which Allen Cognitive level is indicative of continuous neuromuscular adjustments: person may live alone with weekly checks to monitor safety and check problem-solving methods. Person may succeed in supportive employment with a job coach and get to regularly scheduled valued community activity.

4.2 (Differentiating features)

Which Allen Cognitive level is indicative of differentiating features: close supervision to remove dangerous objects outside of the visual field and to solve any problems arising from minor changes in the environment. Person may spend a day allowance, walk to familiar locations in the neighborhood, or follow a simple, familiar bus route.

5.2 (Discriminating Between Parts of an Activity)

Which Allen Cognitive level is indicative of discriminating between parts of an activity: Person may live alone with weekly checks to monitor safety and examine potentially dangerous effects of impulsive behavior. Person may succeed in supportive employment with a job coach and participate in valued community events.

3.0 (grasping objects)

Which Allen Cognitive level is indicative of grasping objects: 24-hour nursing care to elicit habitual motions for activities of daily living and to complete motions for an acceptable level of hygiene.

3.6 (Noting Effects on Objects)

Which Allen Cognitive level is indicative of noting effects on objects close supervision to provide the materials needed for activities of daily living, to remind person to finish necessary steps, to check results, and to remove access to dangerous objects.

2.0 (overcoming gravity)

Which Allen Cognitive level is indicative of overcoming gravity: 24-hour nursing care to transfer from bed to chair, provide food, and and do bathroom activities.

4.6 (Personalizing)

Which Allen Cognitive level is indicative of personalizing: Person may live alone with daily assistance to monitor personal safety. May manage a daily allowance. Bills and other money management concerns require assistance. Person may require reminders to do household chores, to attend familiar community events, or to do anything in addition to daily household routine.

6.0 (planning without objects)

Which Allen Cognitive level is indicative of planning without objects: Person may consider several hypothetical plans of action and establish abstract criteria for selecting the best plan.

2.2 (righting Reactions)

Which Allen Cognitive level is indicative of righting reactions: 24-hour nursing care to prevent standing if unable to weight-bear, transfer on sliding board or a pivot transfer, provide food, and do bathroom activities.

4.8 (Rote learning)

Which Allen Cognitive level is indicative of rote learning: Global cognition is mildly impaired. The person is able to learn new ways of doing things through trial and error problem solving. The person detects the best effect by exploring distinctive properties of objects and trying different actions. They exercise poor judgment with no symbolic thought to plan actions or anticipate potential mistakes. They may make hasty or impulsive decisions or make abrupt changes in their course of action. The determination of what is best may be made according to personal preferences or social standards. The person can imitate a series of new directions; new learning is recognized and repeated during the process of doing an activity.

5.4 (self-directed learning)

Which Allen Cognitive level is indicative of self-directed learning: person may live alone and work in a job with a wide margin of error. Person may not be safe in jobs with a high potential for industrial accidents.

4.0 (Sequencing)

Which Allen Cognitive level is indicative of sequencing: Close supervision to remove dangerous objects and solve any problems occurring through minor changes in routine. Person may fix self a cold meal or snack and make small purchases in the neighborhood.

3.8 (Using All Objects)

Which Allen Cognitive level is indicative of using all objects: close supervision to get materials out that are needed to do activities of daily living, to check results, and to remove dangerous objects.

A (Considered supplemental Medical Insurance Program and must be purchased by the beneficiary, usually as a monthly premium)

Which Medicare plan pays for hospital outpatient physician and other professional services including OT services provided by independent practitioners? Services have no specific time limit and require 20% co-payment. A) Part B B) Part A C) Part D D) Part C

B (Automatically provided to all who are covered by the Social Security System that meet the criteria)

Which Medicare plan pays for inpatient hospital, skilled nursing facility, home health, rehab facilities, and hospice care? A) Part B B) Part A C) Part D D) Part C

Autonomy and confidentiality

Which code of ethics principle is described as OT personnel shall respect the right of the individual to self-determination: establish collaborative relationships with clients/caregivers, etc; obtain consent before administering any OT service; respect the recipient of service's right to refuse; provide students with accurate info; obtain informed consent for research and respect right to withdraw; confidentiality and right to privacy; maintain confidentiality; meaningful communication and comprehension; open collaboration with all involved parties to facilitate comprehension of services and potential risks/benefits?

B (Not all professional dilemmas involve a violation of the Code of Ethics. Before taking action, reflecting on one's own beliefs and values and examining the facts are important. Being able to articulate what the dilemma is usually leads to a better determination of whether a violation is involved. Once the dilemma is clearly framed, it is important to look carefully at the language in the Code to see if it addresses the situation.)

Which activity is an important part of the process of deciding whether a professional dilemma might in fact involve a violation of the Occupational Therapy Code of Ethics (2015)? A. Taking quick action to reduce the potential for the dilemma to become greater B. Clearly stating the dilemma in a way that focuses on the most important issue to be resolved C. Seeking justification for the actions of those involved D. Determining the best way to protect one's self-interest

radial and ulnar

Which arteries are the main arteries supplying blood to the hand and wrist?

C

Which ascending fiber system (sensory pathways) conveys deep and chronic pain to reticular formation of brain stem via diffuse, polysynaptic pathways? A) Dorsal columbs/ Medial Leminscal system B) Spinocerebellar tracts C) Spinoreticular tracts D) Spnothalamic tracts

B

Which ascending fiber system (sensory pathways) conveys proprioception information from muscle spindles, gogi tendon organs, touch, and pressure receptors to cerebellum for control of voluntary movements; dorsal spinocerebellar tract ascends to ipsilateral inferior cerebellar peduncle? A) Dorsal columbs/ Medial Leminscal system B) Spinocerebellar tracts C) Spinoreticular tracts D) Spnothalamic tracts

D

Which ascending fiber system (sensory pathways) conveys sensations of pain and temperature, and cured touch; tracts asced 1 or 2 ipsliateral spinal cord segments, synapse and cross in spinal cord to opposite side and ascend in ventrolateral _____?_______ system? A) Dorsal columbs/ Medial Leminscal system B) Spinocerebellar tracts C) Spinoreticular tracts D) Spnothalamic tracts

A

Which ascending fiber system conveys (sensory pathways) sensations of proprioception, vibration, and tactile discrimination (upper extremity tracts are laterally located, and lower extremity tracts are medially located); neurons ascend to medulla where fibers cross, ascend to thalamus, and then somatosensory cortex? A) Dorsal columbs/ Medial Leminscal system B) Spinocerebellar tracts C) Spinoreticular tracts D) Spnothalamic tracts

B (The Useful Field of View assesses the ability to visually scan information using central and peripheral vision and to process that information in a timely fashion.) (Symbol Digit Modalities Test: This test measures divided attention; Trail Making Tests, Part A and B: These tests measure set shifting, Letter cancellation test: This test measures scanning and selective attention.

Which assessment would an OTR® administer to assess a client's ability to visually scan information using central and peripheral vision and to process that information in a timely fashion? A. Symbol Digit Modalities Test B. Useful Field of View C. Trail Making Test, Parts A and B D. Letter cancellation test

Procedural Justice

Which code of ethics principle is described as OT personnel shall comply with institutional rules, local, state, federal, and international laws and AOTA documents applicable to the profession of OT: be familiar with code of ethics related to work setting and share; familiarize and seek to understand institutional rules and take steps to resolve any ethical conflicts; familiarize revisions in those laws and AOTA policies that apply to the profession of OT; familiarize with established policies and procedures for handling concerns with code of ethics and standards on national, state, local, district, and territorial levels; hold requisite credentials for OT services; high standards of service including CEUs; appropriate supervision to individuals who require it; obtain necessary approvals; no misappropriation of funds; prevent discrimination; help formulate policies; collect fees legally; and maintain ethical principles in business arrangements?

Benefice

Which code of ethics principle is described as OT personnel shall demonstrate a concern for the safety and well-being of the recipients of their services. Therapists conduct evals and re-revals in a timely manner, provide services which are within the therapist's level of competence, continuing education, terminating services when appropriate, report any acts in practice, education, and research that appear unethical or illegal; provide info on code of ethics; and refer to other health care providers when necessary?

Nonmaleficence

Which code of ethics principle is described as OT personnel shall intentionally refrain from actions that cause harm: avoid inflicting harm on others, make every effort to ensure continuity of services, avoid relationships that exploit the recipient of services, students, research participants, etc; no sexual relationships; remedy personal problems and limitations that may cause harm; no drugs, alcohol, etc; must maintain professional boundaries; no compromising client's rights no self fulfilling actions; no bartering for services; and determine proportion of risk to benefit for participants in research study?

Veracity

Which code of ethics principle is described as OT personnel shall provide comprehensive, accurate, and objective info when representing the profession: represent self including credentials honestly; no false, fraudulent, deceptive, misleading, or unfair statements or claims; report in timely manner; all documentation for reimbursement is applicable to laws, guidelines, and regulations; accept responsibility for actions that reduce public trust; honest advertising and marketing; describe type and duration of OT servcies; be honest, fair, accurate, and respectful to others; and give credit where credit is due?

Social Justice Principle

Which code of ethics principle is described as OT personnel shall provide services in a fair and equitable manner: uphold altruistic responsibilities to ensure common good; educate value of OT services; promote activities that benefit health status of community; advocate for just/fair treatment; advocate for recipients of OT services to obtain services needed; understand client factors and their effect on service delivery; consider offering pro bono or sliding scale services?

D

Which form of supervision is characterized as provided only on a needed basis, and may be less than monthly? A) Close B) Routine C) General D) Minimal

behavioral

Which frame of reference relies on the idea that behavior is learned and that it can be unlearned. Using breathing and relaxation techniques during a stressful event can facilitate a change in response.

Fidelity

Which code of ethics principle is described as OT personnel shall treat colleagues and other professionals with respect, fairness, discretion, and integrity: respect traditions, practices, competencies, and responsibilities of their own and other professions; preserve, respect, and safeguard private information about employees, colleagues, students, etc. unless mandated by law; no breaching of code of ethics; resolve perceived institutional violations by utilizing internal resources first; avoid conflicts of interest; avoid using one's position that presents a conflict of interest; use conflict resolution; and be diligent stewards of human, financial, and material resources of their employers?

G-codes

Which codes identify the primary issue being addressed by therapy; modifiers are used to report the person's impairment/limitation/restriction? They are used to track pt. outcomes over time.

B

Which cognitive assessment consists of structured observations of BADL and mobility skills to detect underlying neurobehavioral function? A system of error analysis is utilized to document underlying performance components: neglect, spatial dysfunction, body scheme disorder, apraxia. A. Allen Cognitive Level Test B. A-ONE (Arnadottir OT Neurobehavioral Evaluation) C. Congnistat Neurobehavioral Cognitive Status Exam D. Executive Function Performance Test

A (Discharge is upon entry into, or completion of and educational program or the attainment of a graduate equivalency degree [GED])

Which community based practice setting supports adolescents or adults who require intervention to develop skills that are needed to succeed in secondary and/or post-secondary education? A) Supported education programs B) Schools C) Vocational programs D) Residential programs

C

Which community based practice setting supports people who have prerequisite abilities to work but require training for a specific job and/or ongoing structure, support and/or supervision to maintain employment? A) Supported education programs B) Schools C) Vocational programs D) Residential programs

D

Which community based practice setting supports persons who have a developmental, medical, or psychiatric condition that has resulted in functional deficits that impede independent lving but are not severe enough to require hospitalization? A) Supported education programs B) Schools C) Vocational programs D) Residential programs

B

Which descending fiber systems (motor pathways) arise from primary mtoro cortex, descend in brain stem, cross in medulla via lateral ______?_____ tract to ventral gray matter and are important for voluntary motor control? A) Vestibulospinal tracts B) Corticospinal tracts C) Rubrospinal tracts D) Tectospinal tracts E) Reticulospinal tracts

A

Which descending fiber systems (motor pathways) arise from vestibular nucleus and descend to spinal cord in lateral and medial _______?______ tracts; important for control of muscle tone, antigravilty muscles, and postural reflexes? A) Vestibulospinal tracts B) Corticospinal tracts C) Rubrospinal tracts D) Tectospinal tracts E) Reticulospinal tracts

D

Which descending fiber systems (motor pathways) arises from superior colliculus and descends to ventral gray; assists in head turning in response to visual stimuli? A) Vestibulospinal tracts B) Corticospinal tracts C) Rubrospinal tracts D) Tectospinal tracts E) Reticulospinal tracts

C

Which descending fiber systems (motor pathways) arises in contralateral red nucleus and descends in lateral white columns to spinal gray; assists in motor function? A) Vestibulospinal tracts B) Corticospinal tracts C) Rubrospinal tracts D) Tectospinal tracts E) Reticulospinal tracts

E

Which descending fiber systems (motor pathways) arises in the reticular formation of the brain stem and descends in both ventral and lateral columns, terminates both on dorsal gray and ventral gray? A) Vestibulospinal tracts B) Corticospinal tracts C) Rubrospinal tracts D) Tectospinal tracts E) Reticulospinal tracts

Kleinert

Which flexor tendon protocol requires passive flexion and active extension?

Duran

Which flexor tendon protocol requires passive flexion and passive extension?

D (Must be completed within 48 hours of referral or the person's return home)

Which form of assessment must be used for an initial and comprehensive assessment to verify the person's eligibility for Medicare home health benefits, the continuing need for home care, and to plan for the person's nursing, medical, social, rehab, and discharge needs? A) Comprehensive Rehabilitation Facility Services (CORF) B) Resource Based Relative Value Scale (RBRVS) C) Medicare Certified Provider D) Outcome and Assessment Information Set (OASIS)

A (Massage muscles if craps are severe, replace fluid and electrolytes with fruit juice or a balanced electrolyte drink, loosen clothing and have person lie in cool place)

Which form of heat syndrome is characterized by a normal body temp, nausea, diaphoresis, muscle twitching or spasms, weakness, and/or severe muscle cramps? A) Heat cramps B) Heat exhaustion C) Heat stroke

A (Call 911, lower person's body temp by getting the person to a cooler area, placing ice packs on arterial pressure points, and/or spraying body with a cool mist)

Which form of heat syndrome is characterized by hot, dry red skin; a body temperature higher than 104 degrees; slow deep respiration; tachycardia; dilated pupils, confusion; progressing to seizures and possibly loss of consciousness? A) Heat stroke B) Heat exhaustion C) Heat cramps

B (Replace fluid and electrolytes with fruit juice or a balanced electrolyte drink, loosen clothing and have person lie in cool place)

Which form of heat syndrome is characterized by rapid pulse, decreased blood pressure, nausea, comiting, cool pallid skin, mental confusion, headache, and/or giddiness but no fever? A) Heat stroke B) Heat exhaustion C) Heat cramps

symbolic play

Which form of play is associated with language development?

C

Which form of supervision is characterized as at least monthly direct contact with supervision available as needed by other methods? A) Close B) Routine C) General D) Minimal

B

Which form of supervision is characterized as direct contact at least every two weeks at the site of work, with interim supervision occurring by other methods such as telephone or written communication? A) Close B) Routine C) General D) Minimal

D

Which occupation-based intervention activity is MOST appropriate to develop the bilateral hand skills of a 3-year-old child with autism? A. Holding a cup while pouring water from a large pitcher B. Donning a dress and buttoning the five 2-inch buttons on the dress C. Writing the child's own name while stabilizing the paper D. Holding the handle of a small bucket while filling it with water from a faucet

C (Anyone over 2 years is eligible for this test)

Which of the following ADL/IADL assessments requires the therapist to complete a special training course which includes rater calibration? A. Barthel Index B. FIM and WeeFIN C. AMPS D. Milwaukee Evaluation of Daily Living Skills (MEDLS)

A (Prohibits employment tests that tend to screen out people with disabilities)

Which of the following act allows an employer to ask questions about one's ability to perform a job but prohibits inquiries as to whether one has a disability? A) Americans with Disabilities Act B) Omnibus Budget Reconciliation Act C) Rehabilitation Act of 1973 D) Ticket to Work and Work Incentives Improvement Act (TWIIA)

D (Allows an individual with a disability to keep Medicare benefits for an additional 54 months after starting work. Eliminates Medicaid buy in options)

Which of the following acts astives to make it more realistic and easier for a person with a disability to work, e.g. allowing individuals with disabilities to maintain their Medicare/Medicaid health care benefits, and all states can design their own program? B) Omnibus Budget Reconciliation Act C) Work Investment Act (WIA) D) Ticket to Work and Work Incentives Improvement Act (TWIIA)

C

Which of the following acts is a federally sponsored national employment and vocational training system, it's a one stop center that includes outreach, intake and orientation, initial assessment, eligibility determination for services, assistance with job search and placemnt, job market info and career counseling? A) Americans with Disabilities Act B) Omnibus Budget Reconciliation Act C) Work Investment Act (WIA) D) Ticket to Work and Work Incentives Improvement Act (TWIIA)

C

Which of the following acts offers services for youth (aged 14-21) with disabilities that assist in successful transition from school to work? A) Americans with Disabilities Act B) Omnibus Budget Reconciliation Act C) Rehabilitation Act of 1973 D) Ticket to Work and Work Incentives Improvement Act (TWIIA)

C

Which of the following areas of the brain has the primary auditory cortex: receives and processes auditory stimuli, associative auditory cortex: processes auditory stimuli, and Wernike's area: language comprehension, A. Parietal lobe B. Frontal lobe C. Temporal lobe D. Occipital lobe

A

Which of the following areas of the brain is responsible for sensory integration of sensation, receives fibers conveying touch, proprioceptive, pain and temperature sensations from the opposite side of the body. A. Parietal lobe B. Frontal lobe C. Temporal lobe D. Occipital lobe

B

Which of the following areas of the brain is responsible for voluntary muscle activation, controls emotions, judgments, higher order cognitive functions such as ideation and abstraction, planning of movements, Broca's area (controls motor aspects of speech) A. Parietal lobe B. Frontal lobe C. Temporal lobe D. Occipital lobe

D

Which of the following areas of the brain receives/processes visual stimuli (primary visual cortex), processes visual stimuli (visual association cortex)? A. Parietal lobe B. Frontal lobe C. Temporal lobe D. Occipital lobe

B

Which of the following assessments contains 154 items and assesses performance of real or simulated work tasks based on a person's interests? A. Worker Role Interview B. Smith Physical Capacity Evaluation C. Epic Functional Evaluation System D. Vocational Interest, Temperment, and Aptitude System (VITAS)

D

Which of the following assessments explores, quantifies, and describes performance in central areas of brain-behavior relations, including level of consciousness, orientation, attention, language, constructional ability, memory, calculations, and reasoning? A. Rivermead Behavioral Memory Test B. Lowenstein Occupational Therapy Cognitive Assessment (LOTCA) C. Behavioral Inattention Test D. Cognistat Neurobehavioral Cognitive Status Examination

B

Which of the following assessments is a standardized checklist used to detect the presence and degree of unilateral neglect during observation of everyday life situations? A. Allen Cognitive Level Test B. Catherine Bergego Scale C. Rivermead Behavioral Memory Test D. Assessment of Motor and Process Skills (AMPS)

A

Which of the following disorders is characterized as a person experiences uninterrupted period of illness during which, at some time, there is a major depressive episode, a manic episode, or a mixed episode concurrent with symptomes that meet criterion for schizophrenia? A) Schizoaffective disorder B) Delusional disorder C) Schizophreniform disorder D) Brief psychotic episode

D

Which of the following disorders is characterized as an individual experiences at least one day but less than one month with one or more criterion symptoms of schizophrenia which result from severe psychosocial stress? A) Schizoaffective disorder B) Delusional disorder C) Schizophreniform disorder D) Brief psychotic episode

C

Which of the following disorders is characterized as meeting the criteria for schizophrenia; however, the episode lasts more than one month but less than six months required for a diagnosis of schizophrenia? A) Schizoaffective disorder B) Delusional disorder C) Schizophreniform disorder D) Brief psychotic episode

B

Which of the following disorders is characterized as the individual's predominant sypmtoms are non-bizarre delusions with the absence of other criterion symptoms of schizophrenia? A) Schizoaffective disorder B) Delusional disorder C) Schizophreniform disorder D) Brief psychotic episode

A (The Recovery Model suggests a broad context for intervention, not specific intervention strategies provided by mental health providers. The Recovery Model is a client-centered approach to recovery that encourages the supports necessary for a person to recover within a context that is comfortable for that person. It may include peer support and teaching.)

Which of the following elements is an important component of the Recovery Model? A. Peer support and teaching B. Social-emotional learning C. Cognitive-behavioral therapy D. Sensory-motor interventions

A (Exercise to maintain ROM and strength is important for clients with RA. Although as a general principle AROM is preferred, during a flare-up PROM is indicated if pain makes it difficult for the client to do AROM.)

Which of the following exercise programs is MOST appropriate for a client with rheumatoid arthritis (RA) during an acute flare-up that involves significant pain and swelling? A. Daily PROM of the joints through full comfortable ROM B. Daily AROM of the joints through full comfortable ROM C. Isotonic progressive resistive exercises of the joints as tolerated 3 times a week D. No exercise program

D

Which of the following groups are NOT covered by Medicare? A) Persons 65 and older and retired railroad workers B) Individuals of all ages with end stage renal disease/permanent kidney failure that may require dialysis or kidney transplant C) Persons with a long-term disability who have received government-funded disability benefits for 24 months D) Children whose parents' income is below the poverty level

A

Which of the following is a contraction with movement: eccentric = lengthening and concentric = shortening A) Isotonics B) Isometrics

C

Which of the following is a contraction without movement: sometimes can produce more forceful contraction, are contraindicated for persons with cardiovascular problems; they can increase blood pressure and heart rate, which is why they should be avoided. A) Isotonics B) Contract relax C) Isometrics

B

Which of the following is a result of Omnibus Budget Reconcilation Act (OBRA)? A) Established SSI, which enables persons with disabilities to receive monthly income B) Provided Medicaid financing for community-based services for people with developmental disabilities when services were demonstrated to be less expensive than institutional care. C) Requires all federal agencies develop action plans for hiring, placement, and advancement of persons with disabilities. D) Prohibits discrimination against qualified persons with disabilities in employment, transportation, accommodations, telecommunications, and public services.

C

Which of the following is a result of The rehabilitation Act of 1973? A) Established SSI, which enables persons with disabilities to receive monthly income B) Requires owners of housing to make reasonable exceptions to their standard tenant policies to allow people with disabilities equal housing opportunities C) Requires all federal agencies develop action plans for hiring, placement, and advancement of persons with disabilities. D) Prohibits discrimination against qualified persons with disabilities in employment, transportation, accommodations, telecommunications, and public services.

C

Which of the following prevocational and screening assessments evaluates an idividual's capabilities in relation to one of several dimensions: physical demads of a job, critical demands of a job, critical demands of an occupational group, and the demands of competitive employment? A. Work capacity evaluation B. Job Site analysis C. Functional capacity evaluation

B

Which of the following prevocational and screening assessments evaluates expectations, supports, ergonomics, essential functions of the job, the marginal functions of the job, and the potential resonable accomodations in accordance with ADA? A. Work capacity evaluation B. Job Site analysis C. Functional capacity evaluation

A

Which of the following prevocational and screening assessments evaluates using real or simulated work activities to assess an individual's ability to return to work (e.g. Valpar Work Samples or BTE)? A. Work capacity evaluation B. Job Site analysis C. Functional capacity evaluation

B

Which of the following spinal cord injuries is characterized by an injury at the L1 level and below resulting in the lower motor neuron lesion; flaccid paralysis with no spinal reflex activity; an areflexic bowel and bladder? A. Conus medullaris B. Cauda equina syndrome C. Posterior cord syndrome D. Central cord syndrome

D

Which of the following subtype of schizophrenia is characterized by severe disturbances in motor behavior involving stupor, negativism, rigidity, excitement, or posturing? A) Paranoid type B) Disorganized type C) Undifferentiated type D) Catatonic type

A

Which of the following subtype of schizophrenia is characterized by the preoccupation with one or more delusions of persecution or grandeur. Auditory hallucinations are frequently present; these individuals tend to exhibit fewer negative behaviors? A) Paranoid type B) Disorganized type C) Undifferentiated type D) Residual type

B

Which of the following subtype of schizophrenia is distinguished by marked regression demonstrating primitive, disinhibited, and disorganized behavior? A) Paranoid type B) Disorganized type C) Undifferentiated type D) Residual type

C

Which of the following subtype of schizophrenia is used to classify those patients who do not clearly fit into one of the other categories? A) Paranoid type B) Residual type C) Undifferentiated type D) Catatonic type

B

Which of the following subtype of schizophrenia is used when there is continued evidence of schizophrenic behavior in the absence of a complete set of diagnostic criteria? A) Paranoid type B) Residual type C) Undifferentiated type D) Catatonic type

B (During the functional training phase, a client is learning to incorporate the TD as a functional assist. Bilateral activities that require one dominant extremity (the hand) and one functional assist (the TD) are the best tasks for practice. Using scissors to cut paper requires the TD to be the functional assist (holding the paper)

Which of the following tasks would be MOST appropriate to use during the functional training phase for a client learning to use a unilateral myoelectric terminal device (TD)? A. Grasping and releasing different-sized blocks B. Using scissors to cut paper C. Typing on a computer keyboard D. Brushing teeth

D

Which practice involves repeated performance of the same motor skill? A) Random practice B) Planned Practice C) Contextual practice D) Blocked practice

distal

Which row of carpal bones are made up of the hamate, capitate, trapezoid, and trapezium?

proximal

Which row of carpal bones are made up of the pisiform, lunate, triquetrum, and scaphoid?

D (friends become important for validation of play items and performance, while parents assist and validate in the absence of peers)

Which stage of developmental play does a child begin to participate in cooperative peer groups with a growing interest in competition? Child participates in play with rules, competition, social interaction, and opportunities for development of skills. A. Symbolic Play (2-4 years) B. Creative Play (4-7 years) C. Exploratory play (0-2 years) D. Games (7-12 years)

C

Which stage of developmental play helps a child develop body scheme; sensory integrative and motor skills are also developed as the child explores the properties and effects of actions on objects and people? A. Symbolic Play (2-4 years) B. Creative Play (4-7 years) C. Exploratory play (0-2 years) D. Games (7-12 years)

A (Child mostly involved in parallel play with peers, becomes more cooperative over time)

Which stage of developmental play helps a child develop language skills? Child formulates, tests, classifies, and refines ideas, feelings, and combined actions: objects that are manageable for the child in terms of symbolization, control, and mastery are preferred by the child. A. Symbolic Play (2-4 years) B. Creative Play (4-7 years) C. Exploratory play (0-2 years) D. Games (7-12 years)

C (Child participates in cooperative peer groups.

Which stage of developmental play helps a child explore combinations of actions on multiple objects and master skills that promote performance of school and work related activities? The child engages in sensory, motor, cognitive, and social play experiences in which the child refines relevant skills. A. Games (7-12) B. Exploratory play (0-2 years) C. Creative Play (4-7 years) D. Symbolic Play (2-4 years)

Remodeling

Which stage of fracture healing deposits bone?

Repair

Which stage of fracture healing forms the callus for stabilization?

Inflammation

Which stage of fracture healing provides the cellular activity needed for healing?

C

Which stage of motor learning involves 'carry-over' as individuals are asked to demonstrate their newly acquired skill after initial practice? A) Skill transfer stage (autonomous stage) B) Skill acquisition stage (cognitive stage) C) Skill retention stage (associated stage) D) Generalization

A

Which stage of motor learning involves the individual demonstrating the skill in a new context? A) Skill transfer stage (autonomous stage) B) Skill acquisition stage (cognitive stage) C) Skill retention stage (associated stage) D) Generalization

B

Which stage of motor learning occurs during initial instruction and practice of a skill? A) Skill transfer stage (autonomous stage) B) Skill acquisition stage (cognitive stage) C) Skill retention stage (associated stage) D) Generalization

visual system

Which system develops as an infant responds to human faces and itmes of high contrast place approximately 10 in. from face

Auditory system

Which system is immature at birth and develops as the infant orients to voices and other sounds?

Wartenburg's sign (Can be used to test for cubital tunnel syndrom)

Which test is completed with the fifth finger held away from the fourth finger, indicating palmar adductor interossei weakness.

A (Resources and responsibilities are individually allocated between disciplines; therefore, competition among team members may develop)

Which type of team has a number of professionals from different disiplines who conduct assessments and interventions independent of one another? A) Multidisciplinary B) Intradisciplinary C) Interdisciplinary D) Transdisciplinary

C (Members are directed toward a common goal and not bound by discipline line-specific roles and functions)

Which type of team has all disciplines relevant to the case at hand agree to collaborate for descision making? The exchange of info, prioritization of needs and allocation of resources and responsibilities are based on members' expertise and skills, not on "turf" issues. A) Multidisciplinary B) Intradisciplinary C) Interdisciplinary D) Transdisciplinary

D (Role blurring is accepted. Ongoing training, support, supervision, cooperation, and consultation among disciplines are important to this model, ensuring that professional integrity and quality of care is maintained)

Which type of team has characteristics of interdisciplinary teams and expanded on? Members of this team support and enhance the activities and programs of other disciplines to provide quality, efficient, cost effective service. A) Multidisciplinary B) Intradisciplinary C) Interdisciplinary D) Transdisciplinary

B

Which type of team has one or more members of one discipline who evaluate, plan, and implement treatment of the individual? A) Multidisciplinary B) Intradisciplinary C) Interdisciplinary D) Transdisciplinary

Diabetic retinopathy

Which vision loss pathology results from damage to the retinal capillaries, growth of abnormal blood vessels and hemorrhage leads to scarring and detachment. Individuals generally sustain central vision impairment, blurred vision, blindness is rare.

Macular Degeneration

Which vision loss pathology results from decreased blood supply or abnormal growth of blood vessels under the retina; individuals retain peripheral vision, increased sensitivity to glare, and difficulty adjusting to light change, may progress to total blindness.

Glaucoma

Which vision loss pathology results from increased intraocular pressure, with degeneration of optic disc, atrophy of optic nerve; results in early loss of peripheral vision (tunnel vision)

(CVA) Homonumous hemianiosia (nasal half of one eye, temporal half of other eye)

Which vision loss pathology results in a visual field loss of 1/2 in each eye. Produces an inability to receive information from right or left side; corresponds to side of sensorimotor deficit.

Cataracts

Which vision loss pathology results in gradual loss of vision: central first, then peripheral; increased problem with glare; general darkening of vision; loss of acuity, and distortion?

C (Participants of a sheltered program are paid at a piece-work rate, and participants are paid at the prevailing competitive wage for positions in a TEP and supported employment employment program)

Which work program includes a multidisciplinary or interdisciplinary approach, participants are paid, participants are considered employees with supports provided as needed, a transition between program participation and competitive employment is provided according to participants' functional level, these programs are not required to be accredited but most are part of an accredited hospital system or a major agency? A. Work-hardening B. Work conditioning C. Rehabilitation (sheltered) workshops, supported employment programs, transitional employment programs D. Ergonomic program

A

Which work program includes an interdisciplinary approach, real or simulated work activities are used, a transition between acute care and return to work is provided, and issues of productivity, safety, physical tolerance, and worker behaviors are addressed? A. Work-hardening B. Work conditioning C. Rehabilitation (sheltered) workshops, supported employment programs, transitional employment programs D. Ergonomic program

B

Which work program includes one discipline that provides services, real or simulated work activities are used, a transition between acute care and return to work is provided, flexibility, strength, movement, and endurance are addressed? A. Work-hardening B. Work conditioning C. Rehabilitation (sheltered) workshops, supported employment programs, transitional employment programs D. Ergonomic program

D

Which work program includes prevention as the main focus to fit the work place to the human body, specific job site analysis, a survey, manager and employee training, educational seminars, and exercise and stretching programs? A. Work-hardening B. Work conditioning C. Rehabilitation (sheltered) workshops, supported employment programs, transitional employment programs D. Ergonomic program

C

Who can supervise level I fieldwork students? A) Entry level OTs and seasoned OTs B) OTAs C) All of the above

More

Will an OTA providing services to an acutely ill person with rapidly changing status on an inpaitent unit require more or less of an occupational therapist/OTA partnership than an OT assistant providing services to a more stable client in a long-term care residential facility?

Swan neck deformity

_____________ _____________ ______________ is injury to the MCP, PIP, or DIP joints characterized by PIP hyperextension and DIP flx; the PIP is splinted in slight flx.

sensory, motor

______________ integrative and ___________ skills are developed as the child explores the properties and effects of actions on objects and people.

primitive

_______________ reflexes are present at or just after birth and typically integrate throughout the first year.

Boutonniere deformity

________________ _______________ is the disruption of the central slip of the extensor tendon characterized by PIP flexion and DIP hyperextension; the PIP is splinted in ext. and isolated DIP flx exercises are performed

4-5 months

at what age does munching occur consisting of a phasic bite and release of a soft cookie?

vestibular system

what helps an infant to feel more organized and content? It is also fully developed in the womb but continues to be refined and impacts on the infant's arousal level.

2-3 years

when does digital pronate grasp occur?

4.5 - 6 years

when does dynamic tripod grasp occur?

12 months

when does fine pincer grasp occur?

9 months

when does inferior pincer grasp occur?

two to three years

when does motor planning and praxis ideation progress?

5 months

when does palmar grasp occur?

1-1.5

when does palmar supinate grasp occur?

4 months

when does primitive squeeze grasp occur?

8 months

when does radial digital grasp occur?

6 months

when does radial-palmar grasp occur?

6 months

when does raking occur?

3.5-4 years

when does static tripod grasp occur?

thirteen to twenty-four months

when does symbolic gesturing and vocalization promote ideation, indicating the ability to conceptualize; and motor planning abilities contribute to self concept as the child begins to master the environment?

symbolic play

which form of play do children mostly engage in parallel play with their peers and begins to become more cooperative over time.


Ensembles d'études connexes

unit 2- differentiation definition and fundamental properties

View Set

Los presagios antes de los españoles y el descubrimiento de América

View Set

MAN 4720, Chapter 1, Exam 1 Review

View Set

CSC Healthcare Informatics Exam #2

View Set

Mental Health practice Questions

View Set

Tough, though, thought, through, thorough and throughout

View Set

Geography Trails (South America)

View Set